Complex Care Exam 2 Practice Questions

Pataasin ang iyong marka sa homework at exams ngayon gamit ang Quizwiz!

A nurse is teaching nutritional strategies to a client who has a low serum calcium level and an allergy to milk. Which of the following statements by the client indicates an understanding of the teaching? a. "I will eat more cheese because I can't drink milk." b. "I need to avoid foods with vitamin D because I am allergic to milk." c. "I will stop taking my calcium supplements if they irritate my stomach."" d. I will add broccoli and kale to my diet."

"I will add broccoli and kale to my diet." The nurse should recommend broccoli and kale, which are good sources of calcium as alternatives to milk products.

The nurse provides hypertonic saline (3% NaCL) to a hyponatremic patient with increased intracranial pressure. The nurse understands that hypertonic solution will cause the following effect? A. Fluid shift out of the cells and into the extracellular fluid B. Fluid shift out of the cells and into the interstitial fluid C. Fluid shift out of the extracellular space and into the cells D. Fluid shifts into the cells from the extracellular fluid

A Hypertonic solutions have a greater concentration of solutes than blood. Fluid will move out of the cells and into the extracellular spaces. This fluid movement into the intravascular space decreases cerebral edema thus lessening intracranial pressure.

The nurse is preparing to admit a client to the​ high-acuity unit with a diagnosis of chronic kidney disease. Which restriction should the nurse anticipate for the​ client? (Select all that​ apply.) A.​ Protein-restricted diet B. Fluid restriction C.​ Calorie-controlled diet D.​ Low-residue diet E. DASH diet

A, B

A 78-yr-old patient has stage 3 CKD and is being taught about a low-potassium diet. The nurse knows the patient understands the diet when the patient selects which foods to eat? a. Apple, green beans, and a roast beef sandwich b. Granola made with dried fruits, nuts, and seeds c. Watermelon and ice cream with chocolate sauce d. Bran cereal with ½ banana and milk and orange juice

A

A client involved in a motor vehicle crash presents to the ER with severe internal bleeding. The client is severely hypotensive and unresponsive. The nurse anticipates that which IV solution will most likely be prescribed to increase intravascular volume, replace immediate blood loss volume, and increase blood pressure? A. 5% dextrose in lactated ringers B. 0.33% sodium chloride (1/3 normal saline) C. 0.225% sodium chloride (1/4 normal saline) D. 0.45% sodium chloride (1/2 normal saline)

A

During a head-to-toe assessment on a patient with a possible urinary tract infection, you perform costovertebral angle percussion. The costovertebral angle is found? A. between the bottom of the 12th rib and spine B. between the right upper quadrant and umbilicus C. between the sternal notch and angle of Louis D. between the ischial spine and umbilicus

A

In a severely anemic patient, the nurse would expect to find A. dyspnea and tachycardia. B. cyanosis and pulmonary edema. C. cardiomegaly and pulmonary edema. D. ventricular dysrhythmias and wheezing.

A

The client is being admitted with folic acid deficiency anemia. Which would be the most appropriate referral? A. Alcoholics Anonymous B. Leukemia Society of America C. A hematologist D. A social worker.

A

The nurse anticipates that the pt with hyperphosphatemia secondary to renal failure will require A. calcium supplements B. potassium supplements C. magnesium supplements D. fluid replacement therapy

A

The nurse is caring for a client who is scheduled for the creation of an ileal conduit. Which statement by the client provides evidence that client teaching was effective? A. "My urine will be eliminated through a stoma." B. "I will not need to worry about being incontinent of urine." C. "A catheter will drain urine directly from my kidney." D. "My urine will be eliminated with my feces."

A

The nurse is caring for a client who takes furosemide (Lasix) and digoxin (Lanoxin). The client's potassium (K+) level is 2.5 mEq/L (2.5 mmol/L). Which additional assessment will the nurse make? A. Heart rate B. Blood pressure (BP) C. Increases in edema D. Sodium level

A

The nurse writes a client problem of "activity intolerance" for a client dx with anemia. Which intervention should the nurse implement? A. Pace activities according to tolerance B. Provide supplements high in iron and vitamins C. Administer packed red blood cells D. Monitor vital signs q4h

A

The provider has ordered a nephrostomy tube for a client in the nurse's care. How should the nurse explain the procedure for nephrostomy tube placement to the client? A. The tube is placed directly into the kidney to drain urine B. The tube is placed through the ureter into the renal pelvis C. The tube is placed into renal cortex to facilitate excretion D. The tube is placed in the ureter above the blockage sit

A

When planning care for a client receiving a sulfonamide antibiotic, it is important for the nurse to perform which intervention? A. Encourage fluid intake of 2000 to 3000 mL/day. B. Avoid direct sun exposure and tanning beds. C. Take the medication with dairy products such as milk or yogurt. D. Advise the client to report any tinnitus to the health care provider.

A

Which patient is most at risk for an electrolyte imbalance? A. An 8 month old with a fever of 102.3 'F and diarrhea B. A 55 year old diabetic with nausea and vomiting C. A 5 year old with RSV D. A healthy 87 year old with intermittent episodes of gout

A

Which patient would NOT be a candidate for hypotonic solution? A. A patient with increased intracranial pressure. B. A patient with Diabetic Ketoacidosis. C. Patient experiencing Hyperosmolar Hyperglycemia. D. All of the options are correct.

A

the nurse anticipates that the pt with hyperphosphatemia secondary to renal failure will require A. calcium supplements B. potassium supplements C. magnesium supplements D. fluid replacement therapy

A

When assessing the hemodynamic information for a newly admitted patient in shock of unknown etiology, the nurse will anticipate administration of large volumes of crystalloids when the A. cardiac output is increased and the central venous pressure (CVP) is low. B. pulmonary artery wedge pressure (PAWP) is increased, and the urine output is low. C. heart rate is decreased, and the systemic vascular resistance is low. D. cardiac output is decreased and the PAWP is high.

A A high cardiac output and low CVP suggest septic shock, and massive fluid replacement is indicated. Increased PAWP indicates that the patient has excessive fluid volume (and suggests cardiogenic shock), and diuresis is indicated. Bradycardia and a low systemic vascular resistance (SVR) suggest neurogenic shock, and fluids should be infused cautiously.

A patient with cerebral edema would most likely be order what type of solution? a. 3% Saline b. 0.9% Normal Saline c. Lactated Ringer's d. 0.225% Normal Saline

A A patient with cerebral edema would be ordered a HYPERTONIC solution to decrease brain swelling. The solution would remove water from the brain cells back into the vascular to be excreted. 3% Saline is the only hypertonic option.

NM is a 62 year old woman who is diagnosed with hypercalcemia related to a parathyroid hormone secreting tumor. Other than cancer chemotherapy, a temporary treatment for her hypercalcemia could include administration of: A. acetazolamide B. furosemide C. hydrochlorothiazide D. spironolactone

B

A nurse is caring for a client who has a nasogastric tube attached to low intermittent suctioning. The nurse should monitor for which of the following electrolyte imbalances? A. Hypercalcemia B. Hyponatremia C. Hyperphosphatemia D. Hypomagnesemia

B The nurse should monitor the client for hyponatremia. Nasogastric losses are isotonic and contain sodium.

Which of the following is not a sign of fluid volume overload? A. weight gain B. increased BUN C. polyuria when renal function is normal D. bounding pulses

B decreased (dilution)

The nurse is teaching a client who is taking a potassium-sparing diuretic about precautions while taking this medication. Which of these does the nurse teach the client to avoid or use cautiously? SELECT ALL THAT APPLY. A. Apples B. Bananas C. ACE inhibitors D. Grapes E. Salt substitute

B, C, E

A patient taking trimethoprim-sulfamethoxazole (TMP-SMX) to treat a urinary tract infection complains of a sore throat. The nurse will contact the provider to request an order for which laboratory test(s)? a. Complete blood count with differential b. Throat culture c. Urinalysis d. Coagulation studies

A A sore throat can indicate a life-threatening anemia, so a complete blood count with differential should be ordered.

A nurse is caring for a client who has renal failure and is receiving epoetin. The nurse should monitor the client for which of the following adverse effects? A. Hypertension B. Muscle pain C. Edema D. Dry mouth

A Answer: Hypertension: Epoetin, an erythropoietic growth factor, can cause hypertension. The nurse should monitor the client's BP before and during therapy and inform the provider if it increases.

A patient with septic shock has a BP of 70/46 mm Hg, pulse 136, respirations 32, temperature 104° F, and blood glucose 246 mg/dL. Which of these prescribed interventions will the nurse implement first? a. Give normal saline IV at 500 mL/hr. b. Infuse drotrecogin- (Xigris) 24 mcg/kg. c. Start insulin drip to maintain blood glucose at 110 to 150 mg/dL. d. Titrate norepinephrine (Levophed) to keep mean arterial pressure (MAP) at 65 to 70 mm Hg.

A Because of the low systemic vascular resistance (SVR) associated with septic shock, fluid resuscitation is the initial therapy. The other actions also are appropriate and should be initiated quickly as well.

A patient with septic shock has a urine output of 20 mL/hr for the past 3 hours. The pulse rate is 120 and the central venous pressure and pulmonary artery wedge pressure are low. Which of these orders by the health care provider will the nurse question? A. Give furosemide 40 mg IV. B. Increase normal saline infusion to 150 mL/hr. C. Administer hydrocortisone 100 mg IV. D. Prepare to give drotrecogin alpha 24 mcg/kg/hr.

A Furosemide will lower the filling pressures and renal perfusion further for the patient with septic shock. The other orders are appropriate.

A nurse is assessing a client who has hyperkalemia. The nurse should identify which of the following conditions as being associated with this electrolyte imbalance? A. Diabetic ketoacidosis B. Heart failure C. Cushing's syndrome D. Thyroidectomy

A Hyperkalemia, an increase in serum potassium, is a laboratory finding associated with diabetic ketoacidosis.

A patient in compensated septic shock has hemodynamic monitoring with a pulmonary artery catheter and an arterial catheter. Which information obtained by the nurse indicates that the patient is still in the compensatory stage of shock? a. The cardiac output is elevated. b. The central venous pressure (CVP) is increased. c. The systemic vascular resistance (SVR) is high. d. The PAWP is high.

A In the early stages of septic shock, the cardiac output is high. The other hemodynamic changes would indicate that the patient had developed progressive or refractory septic shock.

The nurse is counseling a patient who will begin taking a sulfonamide drug to treat a urinary tract infection. What information will the nurse include in teaching? a. Drink several quarts of water daily. b. If stomach upset occurs, take an antacid. c. Limit sun exposure to no more than 1 hour each day. d. Sore throat is a common, harmless side effect.

A Patients should drink several quarts of water daily while taking sulfonamides to prevent crystalluria.

Select ALL the signs and symptoms that can present in pernicious anemia: A. Erythema B. Paresthesia of hands and feet C. Racing thoughts D. Extreme hunger E. Depression F. Unsteady gait G. Shortness of breath with activity

B, E, F, G

The nurse is administering a blood transfusion to a client who is hemorrhaging. In which fluid compartment should the nurse identify that the client is experiencing a​ deficit? A. Intravascular fluid B. Transcellular fluid C. Intracellular fluid D. Interstitial fluid

A Rationale: Blood loss causes a deficit in the intravascular fluid​ compartment, which is a subcompartment of extracellular fluid​ (ECF)

A nurse is reviewing the medical records of four clients. Which of the following conditions is a risk factor for chronic pyelonephritis? A. Parkinson's disease B. Diabetes mellitus C. Peptic ulcer disease D. Gallbladder disease

B. Diabetes mellitus A client who has a history of diabetes mellitus is at risk for the development of chronic pyelonephritis due to reduced bladder tone.

A nurse is caring for a client who had dehydration and is receiving IV fluids. Which assessing for complications, the nurse should recognize which of the following manifestations as a sign of fluid overload? a. Increased urine specific gravity b. Hypoactive bowel sounds c. Bounding peripheral pulses d. Decreased respiratory rate

Bounding peripheral pulses Fluid overload results in increased vascular volume and places a greater workload on the heart. Thus, an expected finding is bounding peripheral pulses.

The charge nurse is making assignments on a medical floor. Which client should be assigned to the most experienced nurse? A. The client dx with iron-deficiency anemia who is prescribed iron supplements B. The client dx with pernicious anemia who is receiving vitamin B12 IM C. The client dx with aplastic anemia who has developed pancytopenia D. The client dx with renal disease who has deficiency of erythropoietin

C

What is the goal of the renin-angiotensin-aldosterone system (RAAS)? A. Decrease blood pressure B. Increase the heart rate C. Increase the blood pressure D. Decrease the heart rate

C

You are providing diet teaching to a patient with low iron levels. Which foods would you encourage the patient to eat regularly? A. Herbal tea, apples, and watermelon B. Sweet potatoes, artichokes, and packaged meat C. Egg yolks, beef, and legumes D. Chocolate, cornbread, and cabbage

C

A nurse is caring for a client who is taking filgrastim to treat neutropenia. The nurse should assess the client for which of the following adverse effects? A. Dusky nail beds B. Petechiae C. Enlarged spleen D. Swollen calf

C Answer: Enlarged spleen: With long-term use, filgrastim, a leukopoietic growth factor, can cause an enlarged spleen. The nurse should tell the client to monitor and report abdominal pain or fullness.

Which of the following is a short acting diuretic that has side effects that include hypokalemic metabolic alkalosis & reversible ototoxicity? A. acetazolamide B. chlorothiazide C. furosemide D. spironolactone

C Lasix (trade name) was given the nick name "last's six" (hours) because of its short duration of action. Its a potent loop diuretic.

Patient's blood pressure is: 155/98 What is the patient's MAP? A. 89 mmHg B. 75 mmHg C. 117 mmHg D. 96 mmHg

C MAP = (1/3)155 + (2/3)98 = 117

The nurse is caring for a client who is receiving intravenous (IV) magnesium sulfate. Which assessment parameter is critical? A. Monitoring 24-hour urine output B. Asking the client about feeling depressed C. Assessing the blood pressure hourly D. Monitoring the serum calcium levels

C magnesium is responsible for (1) neuromuscular transmission and cardiovascular tone → if you are giving magnesium → possible hypermagnesemia = - loss of deep tendon reflexes - hypotension secondary to vasodilation - arrythmias, bradycardia, heart blocks - drowsiness

A nurse is caring for a client who is about to begin taking folic acid to treat megaloblastic anemia. The nurse should monitor which of the following laboratory values to determine therapeutic effectiveness? A. Creatinine clearance B. C-reactive protein C. Reticulocyte count D. Amylase level

C reticulocyte count measures the amount of immature RBCs. Folic acid, also called folate, is essential for erythropoiesis. clients who have a folic acid deficiency require a baseline reticulocyte count, as well as serum folate, Hgb, Hct, and RBC count and periodic monitoring during folic acid therapy to determine effectiveness.

A nurse is providing teaching for a client who has venous insufficiency of the lower extremities. Which of the following statements by the client indicates an understanding of the teaching? a. "If my stockings feel tight, I'll just roll them down for a while." b. "I'll put on my elastic stockings at the first sign of swelling." c. "When I sit down to watch television, I'll be sure to put my feet up." d. "It's okay to cross my legs as long as it's for less than an hour."

C "When I sit down to watch television, I'll be sure to put my feet up." Venous insufficiency makes it difficult for blood flow to return to the heart. Elevating the feet will increase the return. The client should elevate them for at least 20 min several times per day.

A nurse is caring for a group of hospitalized clients. Which client is at greatest risk for infection and sepsis? A. 18-year-old who had an emergency splenectomy B. 22-year-old with recently diagnosed sickle cell anemia C. 38-year-old with hemolytic anemia D. 40-year-old alcoholic with liver disease

A Removal of the spleen causes the client to have reduced immune function. Without a spleen, people are less able to remove disease-causing organisms. Sickle cell anemia causes pain and discomfort owing to the changed cell morphology. Acute pain, especially at joints, is the greatest threat to this client. A low red blood cell (RBC) count can contribute to a client's risk for infection, but this client is more at risk for low oxygen levels and ensuing fatigue.The liver plays a role in blood coagulation. This client is more at risk for coagulation problems than for infection.

A nurse on a critical care unit is caring for a client who has shallow and rapid respirations, paradoxical pulse, CVP 4 cm H2O, BP 90/50 mm Hg, skin cold and pale, and urinary output 55 mL over the last 2 hr. From these findings, the nurse concludes that he may be developing which of the following? A. Hypovolemic Shock B. Cardiac Tamponade C. Sepsis D. Atelectasis

A The client's signs and symptoms are all indicative of hypovolemic shock. The nurse should conclude that the client may be developing this outcome BP makes it hypovolemic shock and not sepsis

The patient has rapidly progressing glomerular inflammation. Weight has increased and urine output is steadily declining. What is the priority nursing intervention? a. Monitor the patient's cardiac status. b. Teach the patient about hand washing. c. Obtain a serum specimen for electrolytes. d. Increase direct observation of the patient.

A The nurse's priority is to monitor the patient's cardiac status. With the rapidly progressing glomerulonephritis, renal function begins to fail and fluid, potassium, and hydrogen retention lead to hypervolemia, hyperkalemia, and metabolic acidosis. Excess fluid increases the workload of the heart, and hyperkalemia can lead to life-threatening dysrhythmias.

A nursing student is observing his or her preceptor provide a hypotonic solution to a patient for dehydration. The preceptor educates the nursing student that hypotonic solutions can cause fluid shifts into the intracellular space for cellular rehydration. What percentage of body fluid is found in the intracellular fluid? A. 70% B. 50% C. 30% D. 15%

A The total body fluid contained within the cells is 70%. The extracellular fluid makes up the remaining 30% of total body fluid.

A nurse is caring for a client who has a serum sodium level 133 mEq/L and serum potassium level 3.4 mEq/L. The nurse should recognize that which of the following treatments can result in these laboratory findings? A. Three tap water enemas B. 0.9% sodium chloride solution IV at 50 mL/hr C. 5% dextrose in water solution with 20 mEq of K+ IV at 80 mL/hr D. Antibiotic therapy

A Three tap water enemas can result in a decrease in serum sodium and potassium. Tap water is hypotonic, and gastrointestinal losses are isotonic. This creates an imbalance and solute dilution.

A nurse should assess a client who has megaloblastic anemia for indications of which of the following vitamin deficiencies? A. Vitamin B12 B. Vitamin C C. Calcium D. Vitamin E

A Vitamin B12: Clients who have megaloblastic anemia have a deficiency of vit b12, folic acid, or both. Cyanocobalamin (vit b12) treats moderate deficiencies. clients who have severe deficiency should take cyanocobalamin and folic acid.

A patient has a folic acid deficiency related to chronic alcohol abuse. The nurse would expect a complete blood cell count (CBC) to reveal A. macrocytic, normochromic red cells. B. normocytic, normochromic red cells. C. microcytic, hypochromic red cells. D. microcytic, normochromic red cells.

A With folic acid deficiency, the cells are larger than normal, but the iron levels are normal or elevated, leading to findings of a macrocytic, normochromic anemia. Microcytic anemia, hypochromic anemia is more typical of iron deficiency. Normocytic, normochromic RBC indicate that the patient does not have anemia or may occur in patients with anemia-related chronic disease.

The nurse is admitting a 24 year old American American female client with a dx of rule-out anemia. The client has a hx of gastric bypass surgery for obesity 4 years ago. Current assessment findings include height 5'5, wt. 75 kg, P 110, R 27, and BP 104/66; pale mucous membranes and dyspnea on exertion. Which type of anemia would the nurse suspect the client has developed? A. Vitamin B12 deficiency B. Folic acid deficiency C. Iron deficiency D. Sickle cell anemia

A stomach produces intrinsic factor = vitamin b12

A nurse is providing teaching for a client who has chronic kidney disease (CKD). Which of the following client statements indicates an understanding of the teaching? A. "I will monitor my blood pressure on the same day each week." B. "I will take milk of magnesia if I'm constipated." C. "I will weigh myself each morning." D. "I will use a salt substitute in my diet."

C. "I will weigh myself each morning." The client who has CKD should monitor his weight every morning at the same time to provide an accurate assessment of fluid balance.

A nurse assessing a client determines that he is in the compensatory stage of shock. Which of the following findings support this conclusion? A. Confusion B. Lethargy C. Unconsciousness D. Petechiae

A Rationale: Confusion characterizes the compensatory stage of shock, as do decreased urinary output, cold and clammy skin, and respiratory alkalosis B. NO - progressive C. NO - irreversible D. NO - progressive straight from ATI book

The nurse is caring for a client who is experiencing insensible water losses. Which of the following is not considered an insensible water loss? (Select all that apply) a. Urine b. Feces c. Respirations d. Diaphoresis

A, B

The pt has a serum phosphate level of 4.7 mg/dL. Which interdisciplinary treatments would the nurse expect for this pt? Select all that apply. A. IV normal saline B. calcium containing antacids C. IV potassium phosphate D. encouraging milk intake E. increasing vitamin D intake

A, B Serum phosphate level of 4.7 mg/dL indicates hyperphosphatemia. IV normal saline promotes renal excretion of phosphate.

A nurse is planning care for a client who has postrenal AKI due to metastatic cancer. The client has a serum creatinine of 5 mg/dL. Which of the following interventions should the nurse include in the plan? (Select all that apply.) A. Provide a high‐protein diet. B. Assess the urine for blood. C. Monitor for intermittent anuria. D. Weight the client once per week. E. Provide NSAIDs for pain.

A, B, C A. CORRECT: The nurse should provide a high‐protein diet due to the high rate of protein breakdown that occurs with acute kidney injury. B. CORRECT: The nurse should assess urine for blood, stones, and particles indicating an obstruction of the urinary structures that leave the kidney. C. CORRECT: The nurse should assess for intermittent anuria due to obstruction or damage to kidneys or urinary structures. D. The nurse should weigh the client daily to monitor for fluid retention due to acute kidney injury. E. The nurse should not administer NSAIDs, which are toxic to the nephrons in the kidney.

A client with hypokalemia has a prescription for parenteral potassium chloride (KCl). Which of these interventions does the nurse use to safely administer KCl? SELECT ALL THAT APPLY. A. Use a potassium infusion prepared by a registered pharmacist. B. Assess for burning or redness during infusion. C. Infuse at a rate of no more than 10 mEq per hour. D. Administer only through a central venous catheter. E. Administer by IV push only during cardiac arrest.

A, B, C RATIONALE: Interventions to safely administer KCl to a client with hypokalemia include: using a pharmacy prepared potassium infusion, checking the client for any burning or redness during infusion, and infusing the IV at not more than 10 mEq per hour. The Joint Commission's National Client Safety Goals mandates that concentrated potassium be diluted and added to IV solutions only in the pharmacy by a registered pharmacist and that vials of concentrated potassium not be available in client care areas. IV potassium solutions irritate veins and cause phlebitis. Assess the IV site hourly, and ask the client whether he or she feels burning or pain at the site. The presence of pain or burning at the insertion site may require a new intravenous to be started. A dose of KCl 5-10 mEq/hour, no more than 20 mEq/hr is recommended.Potassium may be administered by peripheral or central vein. There is no circumstance where potassium is given by IV push.

Which of the following symptoms would be expected in the patient diagnosed with Polycythemia Vera? A. Headache B. Palpable spleen C. Angina D. Polyuria E. Dyspnea

A, B, C, E

A nurse is planning care for a client who has Stage 4 chronic kidney disease. Which of the following actions should the nurse include in the plan of care? (Select all that apply.) A. Assess for jugular vein distention. B. Provide frequent mouth rinses. C. Auscultate for a pleural friction rub. D. Provide a high‐sodium diet. E. Monitor for dysrhythmias.

A, B, C, E B is correct because the nurse should provide frequent mouth rinses due to uremic halitosis caused by urea waste in the blood.

A​ high-acuity client has a suspected acute kidney injury. Which laboratory test should the nurse anticipate for the​ client? (Select all that​ apply.) A. Uric acid B. Creatinine C. Antinuclear antibodies D. Blood urea nitrogen E. Magnesium

A, B, D

A nurse is teaching a female client who has a severe UTI about ciprofloxacin. Which of the following information about adverse reactions should the nurse include? (Select all that apply) a. Observe for pain and swelling of the Achilles tendon b. Watch for a vaginal yeast infection c. Expect excessive nighttime perspiration d. Inspect the mouth for cottage cheese-like lesions e. Take the medication with a dairy product

A, B, D Pain and swelling of the Achilles tendon is an adverse reaction to this medication. A vaginal yeast infection and the cottage cheese-like lesions in the mouth both are caused by overgrowth of Candida albicans and are common reactions to the medication.Dairy products have calcium that reduces the effect of ciprofloxacin. Clients should take this medication 6 hours before or 2 hours after eating dairy products.

A patient is being evaluated for iron-deficiency anemia. Which of these laboratory results would the healthcare provider expect? Select all that apply. A. Decreased serum ferritin B. Decreased hematocrit C. Increased mean corpuscular volume (MCV) D. Increased total iron binding capacity (TIBC) E. Increased hemoglobin

A, B, D The red blood cells of a patient with iron-deficiency anemia tend to be small and pale. Ferritin is an iron-storage protein that will be decreased in iron-deficiency anemia. An increase in the TIBC is an indication that transferrin (an iron transport protein) is carrying less iron because less iron is available

A nurse is admitting a client who reports nausea, vomiting, and weakness. The client has dry oral mucous membranes. Which of the following findings should the nurse identify as manifestations of fluid volume deficit? (select all that apply) A. Decreased skin turgor B. Concentrated urine C. Bradycardia D. Low-grade fever E. Tachypnea

A, B, D, E

A nurse is admitting an older adult client who is experiencing dyspnea, weakness, weight gain of 2lbs, and 1+ bilateral edema of the lower extremities. The client has temperature 99 degrees F, pulse 96/min, respirations 26/min, oxygen saturation 94% on 3L via nasal cannula, and blood pressure 152/96mmHg. Which of the following manifestations of fluid volume excess should the nurse expect? (select all that apply) A. Dyspnea B. Edema C. Bradycardia D. Hypertension E. Weakness

A, B, D, E

A nurse is caring for several clients. Which of the following clients are at risk for developing pyelonephritis? (Select all that apply) a. a client who is at 32 weeks gestation b. a client who has a kidney calculi c. a client who has a urine pH of 4.2 d. a client who has a neurogenic bladder e. a client who has diabetes mellitus

A, B, D, E

Patients with pyelonephritis often have: a. fever b. chills c. burning with bowel movements d. CVA tenderness e. nausea and vomiting

A, B, D, E

The nurse working with a patient after an ileal conduit notices that the pouching system is leaking small amounts of urine. The appropriate nursing intervention is which of the following? A. Empty the pouch. B. Secure/patch it with tape. C. Secure/patch with barrier paste. D. Change wafer and pouch.

D Whenever the nurse notes a leaking pouching system, the nurse should change the wafer and pouch. Attempting to secure or patch the leak with tape and/or barrier paste will trap urine under the barrier or faceplate, which will compromise peristomal skin integrity. Emptying the pouch will not rectify the leaking

The nurse preceptor is teaching a new graduate about conditions that can cause damage to the renal parenchyma and nephrons resulting in acute kidney injury​ (AKI). Which condition should the nurse preceptor​ include? (Select all that​ apply.) A. Glomerulonephritis B. Hemolysis C. Dehydration D. Hypertension E. Vasculitis

A, B, D, E C is prerenal (hypoperfusion issue)

When administering oral erythromycin to a client who has acute diphtheria, a nurse should monitor for which of the following adverse effects? A. Hypothermia B. Blurred vision C. Constipation D. Cardiac dysrhythmias

D. Cardiac dysrhythmias

A nurse is providing instructions regarding reduced dietary intake of potassium for a client who has chronic kidney disease. Which of the following food selections is appropriate for the nurse to recommend to the client? A. 1 cup cubed cantaloupe B. 1 cup boiled spinach C. One baked potato D. One large apple

D. One large apple Of the listed foods, one large apple is the lowest in potassium, containing 239 mg per serving.

A nurse is providing teaching for a client who is at risk for developing respiratory acidosis following surgery. Which of the following statements by the client indicates an understanding of the teaching? a. "I should conserve energy by limiting my physical activity." b. "I will wait until my pain is at least six out of ten before I use the PCA." c. "I will limit my daily fluid intake to two to three glasses." d. "I will use the incentive spirometer every hour."

"I will use the incentive spirometer every hour." Respiratory depression and limited chest expansion are both causes of respiratory acidosis. Using an incentive spirometer will promote adequate chest expansion. The nurse should identify this statement as indicating an understanding of the teaching.

A nurse is planning care for a client who has experienced excessive fluid loss. Which of the following interventions should the nurse include in the plan of care? a. Administer IV fluids to the client evenly over 24 hr. b. Provide the client with a salt substitute. c. Assess the client for pitting edema. d. Encourage the client to rise slowly when standing up. e. Weigh the client every 8 hr.

-Administer IV fluids to the client evenly over 24 hr -Encourage the client to rise slowly when standing up -Weigh the client every 8 hr Administer IV fluids to the client evenly over 24 hr is correct. A client who has excessive fluid loss is typically prescribed IV replacement fluids. Administering IV fluids rapidly over a short period of time places the client at risk for fluid volume overload. Provide the client with a salt substitute is incorrect. There is no reason to limit the client's sodium intake. The client might require electrolyte replacement, depending on the cause of fluid loss. Assess for pitting edema is incorrect. This action is appropriate for a client who has fluid overload. Encourage the client to rise slowly when standing up is correct. This action can prevent injury from falls caused by orthostatic hypotension. Weigh the client every 8 hr is correct. Weighing the client every 8 hr will provide information regarding fluid balance.

A nurse is caring for a client who has a sodium level of 155 mEq/L. Which of the following IV fluids should the nurse anticipate the provider to prescribe? a. Dextrose 5% in 0.9% sodium chloride b. Dextrose 5% in lactated Ringer's c. 3% sodium chloride d. 0.45% sodium chloride

0.45% sodium chloride A sodium level of 155 mEq/L is an indication of hypernatremia. The nurse should anticipate a prescription for a hypotonic solution. The 0.45% sodium chloride is a hypotonic solution used to provide free water and treat cellular dehydration, which promotes waste elimination by the kidneys.Dextrose 5% in 0.9% sodium chloride is a hypertonic solution. The 3% sodium chloride is a hypertonic solution. Lactated Ringer's solution contains sodium and other electrolytes and is not indicated for hypernatremia.

A nurse is admitting a client who takes 40 mg furosemide daily for heart failure and has experienced 3 days of vomiting. The nurse suspects hypokalemia. Which of the following medications should the nurse prepare to administer? a. Sodium polystyrene sulfonate 30 g/day b. 0.9% sodium chloride with 10 mEq/L of potassium chloride at 100 mL/hr c. Bumetanide 8 mg/day d. 100 mL of dextrose 10% in water with 10 units of insulin

0.9% sodium chloride with 10 mEq/L of potassium chloride at 100 mL/hr This IV solution will provide adequate fluid and potassium replacement to offset the losses from vomiting. The typical amount of potassium chloride to administer IV is 5 to 10 mEq/hr and not to exceed 20 mEq/hr. The dilution should be 1 mEq to 10 mL of 0.9% sodium chloride.

A nurse is providing dietary teaching to a client who has heart failure and is receiving furosemide. Which of the following foods should the nurse recommend as containing the greatest amount of potassium? a. 1/2 cup chopped celery b. 1 cup plain yogurt c. 1 slice whole grain bread d. 1/2 cup cooked tofu

1 cup plain yogurt One cup of plain yogurt contains 380 g of potassium. Therefore, the nurse should recommend this food as containing the greatest amount of potassium.

A client with diarrhea for 3 days and inability to eat or drink well is brought to the emergency department (ED) by her family. She states she has been taking her diuretics for congestive heart failure (CHF). What nursing actions are indicated at this time? SELECT ALL THAT APPLY. A. Place the client on bed rest. B. Evaluate the electrolyte levels. C. Administer the ordered diuretic. D. Assess for orthostatic hypotension E. Initiate cardiac monitoring.

A, B, D, E RATIONALE: Nursing actions indicated at this time include: placing the client on bedrest and assisting the client out of bed, evaluating electrolyte levels, assessing for orthostatic hypotension, and applying a cardiac monitor. Safety is required to prevent falls due to weakness from a likely fluid volume deficit and electrolyte imbalance. The nurse should review the laboratory and diagnostic results to detect likely loss of sodium, potassium, and magnesium secondary to diarrhea and diuretic us. Fluid volume deficit is likely with diarrhea and diuretic use and leads to fluid and electrolyte imbalances, especially hypokalemia. Assessing for orthostatic changes will confirm presence of volume deficit. Monitoring for inverted T wave or presence of U wave on the ECG as well as dysrhythmias is indicated when hypokalemia is anticipated.Diuretics increase loss of fluids and electrolytes. The nurse would question this order in the presence of assessment data indicating fluid loss from the diuretics and diarrhea.

A patient that experiences prolonged bleeding is diagnosed with thrombocytopenia. The nurse expects what assessment findings? Select all that apply. A. Fainting B. Dizziness C. Bradycardia D. Hypertension E. Abdominal pain

A, B, E A patient with thrombocytopenia will experience fainting, dizziness, and abdominal pain because of prolonged bleeding. A patient with thrombocytopenia will have tachycardia and hypotension because of decreased quantity of blood.

Your patient, who is post-op from a gastrointestinal surgery, is presenting with a temperature of 103.6 'F, heart rate 120, blood pressure 72/42, increased white blood cell count, and respirations of 21. An IV fluid bolus is ordered STAT. Which findings below indicate that the patient is progressing to septic shock? Select all that apply: A. Blood pressure of 70/34 after the fluid bolus B. Serum lactate less than 2 mmol/L C. Patient needs Norepinephrine to maintain a mean arterial pressure (MAP) greater than 65 mmHg despite fluid replacement D. Central venous pressure (CVP) of 18

A, C To know if the patient is progressing to septic shock, you need to think about the hallmark findings associated with this condition. Septic shock is characterized by major persistent hypotension (<90 SBP) that doesn't respond to IV fluids (refractory hypotension), and the patient needs vasopressors (ex: Norepinephrine) to maintain a mean arterial pressure greater than 65 and their serum lactate is greater than 2 mmol/L. A serum lactate greater than 2 indicates the cell's tissue/organs are not functioning properly due to low oxygen; hence tissue perfusion is poor due to the low blood pressure and mean arterial pressure.

A nurse is planning care for a client who has chronic pyelonephritis. Which of the following actions should the nurse plan to take? (Select all that apply) a. provide a referral for nutrition counseling b. encourage daily fluid intake of 1 L c. palpate the costovertebral angle d. monitor urinary output e. administer antibiotics

A, C, D, E

A nurse is preparing educational material to present to a female client who has frequent urinary tract infections. Which of the following information should the nurse include? (select all that apply) a. avoid sitting in a wet bathing suit b. wipe the perineal area back to front following elimination c. empty the bladder when there is an urge to void d. wear synthetic fabric underwear e. take a shower daily

A, C, E

A nurse is caring for a 56-year-old male patient diagnosed with bladder cancer who has a urinary diversion. Which actions would the nurse take when caring for this patient? Select all that apply a. Measure the patient's fluid intake and output. b. Keep the skin around the stoma moist. c. Empty the appliance frequently. d. Report any mucous in the urine to the primary care provider. e. Encourage the patient to look away when changing the appliance. f. Monitor the return of intestinal function and peristalsis.

A, C, F When caring for a patient with a urinary diversion, the nurse should measure the patient's fluid intake and output to monitor fluid balance, change the appliance frequently, monitor the return of intestinal function and peristalsis, keep the skin around the stoma dry, watch for mucous in the urine as a normal finding, and encourage the patient to participate in care and look at the stoma.

Which of the following are potential causes of hypocalcemia? A. alcoholism B. immobilization C. adrenal insufficiency D. long-term steroid use E. pancreatitis F. furosemide

A, D, E, F NO - B, C = causes of hypercalcemia - C would be correct if it was renal insufficency

You receive a patient in the ER who has sustained a cervical spinal cord injury. You know this patient is at risk for neurogenic shock. What hallmark signs and symptoms, if experienced by this patient, would indicate the patient is experiencing neurogenic shock? Select all that apply: A. Blood pressure 69/38 B. Heart rate 170 bpm C. Blood pressure 250/120 D. Heart rate 29 E. Warm and dry extremities F. Cool and clammy extremities G. Temperature 104.9 'F H. Temperature 95 'F

A, D, E, and H. Hallmark signs and symptoms of neurogenic shock are: - hypotension - bradycardia - hypothermia - warm/dry extremities (this is due to the vasodilation and blood pooling and will be found in the extremities).

A nurse is teaching a client about taking ferrous sulfate to treat iron-deficiency anemia. Which of the following instructions should the nurse include? A. Eat iron-enriched foods B. Spread the dosage across each day C. Take the drug on an empty stomach D. Increase dietary fiber E. report dark green/black stools

A,B,C,D Eat iron-enriched foods is correct. A client who has iron-deficiency anemia should increase iron intake by eating foods such as egg yolks, wheat germ, meat, and fish.Spread the dosage across each day is correct. Spreading out the iron intake throughout the client's waking hours allows the bone marrow to maximize the production of RBCs.Take the drug on an empty stomach is correct. Food reduces the absorption of ferrous sulfate. The client should take the drug on an empty stomach to increase drug absorption. If GI effects are troublesome, they can take the drug with food.Report dark green or black stools is incorrect. The nurse should tell the client to expect dark green or black stools. However, it is not necessary to report this adverse effect.Increase dietary fiber intake is correct. Ferrous sulfate can cause constipation. The client should increase fiber and fluid intake and exercise more often or more intensely.

A nurse is providing dietary teaching to a client who has kidney disease. Which of the following food choices should the nurse include in the teaching as containing the lowest amount of magnesium? a. 1 large hard-boiled egg b. 1 cup bran cereal c. 1/2 cup almond d. 1 cup cooked spinach

A. 1 large hard-boiled eggs One large hard-boiled egg contains 5 mg of magnesium. Therefore, the nurse should recommend this food as containing the lowest amount of magnesium.Cereal has 112 mg. Almonds 193 mg and spinach 157 mg.

A nurse is planning care for a group of clients. Which of the following clients should the nurse plan to monitor for signs of nephrotoxicity? A. A client who is receiving gentamicin for treatment of a wound infection B. A client who is receiving digoxin for treatment of heart failure C. A client who is receiving methylprednisolone for treatment of severe asthma D. A client who is receiving propranolol for treatment of hypertension

A. A client who is receiving gentamicin for treatment of a wound infection Aminoglycoside antibiotics can injure cells of the proximal renal tubules, causing acute tubular necrosis. The nurse should plan to monitor this client for nephrotoxicity and acute kidney injury.

A nurse is reviewing the laboratory reports of a client who has acute kidney injury (AKI). Which of the following findings should the nurse expect? (Select all that apply.) A. BUN 30 mg/dL B. Urine output of 40 mL in past 3 hr C. Potassium 3.6 mEq/L D. Serum calcium 9.8 mg/dL E. Hematocrit 30%

A. BUN 30 mg/dL B. Urine output of 40 mL in past 3 hr E. Hematocrit 30% A. BUN 30 mg/dL is correct. This BUN level is elevated, which is an expected finding for a client who has AKI. B. Urine output of 40 mL in past 3 hr is correct. Oliguria with a urine output 100 to 400 mL per 24 hr is an expected finding for a client who has AKI. E. Hematocrit 30% is correct. This hematocrit level is decreased, which is an expected finding for a client who has AKI.

A nurse is caring for a client who is taking warfarin and has a new prescription for trimethoprim/sulfamethoxazole to treat a urinary tract infection. The nurse should clarify the prescriptions with the provider because taking these two drugs concurrently can increase the client's risk for which of the following? A. Bleeding B. Thrombosis C. ECG changes D. Ototoxicity

A. Bleeding

The RN is assessing a 70-year-old client admitted to the unit with severe dehydration. Which finding requires immediate intervention by the nurse? A. Client behavior that changes from anxious to lethargic B. Deep furrows on the surface of the tongue C. Poor skin turgor with tenting remaining for 2 minutes after the skin is pinched D. Urine output of 950 mL for the past 24 hours

A. Client behavior that changes from anxious to lethargic RATIONALE: Immediate intervention by the nurse is required when a client's behavior changes from anxious to lethargic. This change in mental status suggests poor cerebral blood flow and fluid shifts within the brain cells. Immediate intervention is needed to prevent further cerebral dysfunction.Deep furrows on the surface of the tongue, poor skin turgor, and low urine output are all caused by the fluid volume deficit, but do not indicate complications of dehydration that are immediately life-threatening.

A nurse is assessing a client who has hypomagnesemia. Which of the following findings should the nurse expect? a. Hyperactive deep-tendon reflexes b. Increased bowel sounds c. Drowsiness d. Decreased blood pressure

A. Hyperactive deep-tendon reflexes remember ↓ mg = ↓ ca+ = ↑ phos symptoms - muscle cramps - paresthesia - hyperactive reflexes - tetany

A nurse is performing an admission assessment on a client who has severe chronic kidney disease (CKD). Which of the following findings should the nurse expect for this client? A. Tachypnea B. Hypotension C. Exophthalmos D. Insomnia

A. Tachypnea The nurse should expect the client who has severe CKD to have tachypnea due to metabolic acidosis.

A nurse in a provider's office receives a call from a client who is taking ciprofloxacin to treat a respiratory tract infection and reports dyspepsia. Which of the following instructions should the nurse give the client? A. Take an antacid at least 2 hr after taking the drug. B Take the drug with a cup of coffee. C. Take an iron supplement with the drug. D. Take the drug with 240 mL (8 oz) of milk.

A. Take an antacid at least 2 hr after taking the drug.

A nurse working in the emergency department is caring for a client who reports costovertebral angle tenderness, nausea, and vomiting. For which of the following laboratory values should the nurse notify the provider? A. WBC 15,000/mm3 B. BUN 15 mg/dL C. Urine specific gravity 1.020 D. Urine pH 5.5

A. WBC 15,000/mm3 The WBC count is above the expected reference range and indicates the presence of an infection. The nurse should report this laboratory value to the provider. Normal range 4,000-11,000 mm3

A nurse is planning care for a client who has acute glomerulonephritis. The nurse should plan to provide which of the following interventions? A. Weigh the client daily B. Encourage the client to drink 2 to 3 L of fluid per day. C. Instruct the client to ambulate every 2 hr. D. Obtain the client's serum blood glucose.

A. Weight the client daily The nurse can monitor fluid retention by weighing the client daily.

A nurse is caring for a client who requires nasogastric suctioning. Which of the following set of laboratory results indicates that the client has metabolic alkalosis? a. pH 7.51, PaO2 94 mm Hg, PaCO2 36 mm Hg, HCO3- 31 mEq/L b. pH 7.48, PaO2 89 mm Hg, PaCO2 30 mm Hg, HCO3- 26 mEq/L c. pH 7.31, PaO2 77 mm Hg, PaCO2 52 mm Hg, HCO3- 23 mEq/L d. pH 7.26, PaO2 84 mm Hg, PaCO2 38 mm Hg, HCO3- 20 mEq/L

A. pH 7.51, Pa02 94 mm Hg, PaC02 36 mm Hg, HCO3- 31 mEq/L An elevated pH and HCO3- with a PaCO2 within the expected reference range indicates metabolic alkalosis.

A 42-year-old male patient is admitted with a spinal cord injury. The patient is experiencing severe hypotension and bradycardia. The patient is diagnosed with neurogenic shock. Why is hypotension occurring in this patient with neurogenic shock? A. The patient has an increased systemic vascular resistance. This increases preload and decreases afterload, which will cause severe hypotension. B. The patient's autonomic nervous system has lost the ability to regulate the diameter of the blood vessels and vasodilation is occurring. C. The patient's parasympathetic nervous system is being unopposed by the sympathetic nervous system, which leads to severe hypotension. D. The increase in capillary permeability has depleted the fluid volume in the intravascular system, which has led to severe hypotension.

B

A client's kidneys are retaining increased amounts of sodium. The nurse plans care, anticipating that the kidneys also are retaining greater amounts of which substances? A. Calcium and Chloride B. Chloride and bicarbonate C. Potassium and Phosphates D. Aluminum and magnesium

B

A patient who will begin taking trimethoprim-sulfamethoxazole (TMP-SMX) asks the nurse why the combination drug is necessary. The nurse will explain that the combination is used to a. broaden the antibacterial spectrum. b. decrease bacterial resistance. c. improve the taste. d. minimize toxic effects.

B

A patient with a urinary tract infection is taking Sulfamethoxazole/Trimethoprim. As the nurse you know it is important that the patient consumes 2.5 to 3 L of fluid per day to prevent which of the following complications? A. Brown urine B. Crystalluria C. Renal Stenosis D. Renal Calculi

B

A patient with severe systolic heart failure (very low ejection fraction) and subsequent decreased renal function & low GFR may sometimes fail to respond to a loop diuretic alone (a situation commonly referred to as diuretic resistance). In this case, adding an additional diuretic that acts on an different portion of the nephron can produce a more potent diuresis. The class of diuretic that can be added to a loop diuretic to produce a more potent diuresis is? A. acetazolamide B. chlorothiazide C. furosemide D. spironolactone

B

An older adult​ high-acuity client has a history of​ long-term non-steroidal​ anti-inflammatory drug use. The nurse should monitor the client for which type of kidney​ injury? A. Postrenal kidney injury B. Prerenal kidney injury C. Acute tubular necrosis D. Endogenous intrinsic kidney injury

B

During the __________ stage of shock, the signs and symptoms are very subtle. However, cells are experiencing _________ due to the lack of tissue perfusion, which causes the cells to switch from ___________ metabolism to _________ metabolism. A. Proliferative, hyperoxia, anaerobic, aerobic B. Initial, hypoxia, aerobic, anaerobic C. Compensatory, hypoxia, anaerobic, aerobic D. Fibrotic, hypoxia, aerobic, anaerobic

B

Patient's blood pressure is: 160/102 What is the patient's MAP and how do you interpret this finding? A. 100 mmHg, normal B. 121 mmHg, high C. 65 mmHg, normal D. 86 mmHg, low

B

The physician orders a patient with suspected iron-deficiency anemia a blood smear test to assess the quality of the red blood cells. How would the red blood cells appear if the patient had iron- deficiency anemia? A. Hyperchromic and macrocytic B. Hypochromic and microcytic C. Hyperchromic and macrocytic D. Hypochromic and macrocytic

B

Tissue injury can cause an increase in white blood cell recruitment. The majority of the body's water is contained in which of the following fluid compartments? A. Interstitial B. Intracellular C. Extracellular D. Intravascular

B

What type of fluid would a patient with severe hyponatremia most likely be started on? A. Hypotonic B. Hypertonic C. Isotonic D. Colloid

B

When administering a hypertonic solution the nurse should closely watch for? A. Signs of dehydration B. Pulmonary Edema C. Fluid volume deficient D. Increased Lactate level

B

When assessing the patient who has a lower urinary tract infection (UTI), the nurse will initially ask about a. flank pain. b. pain with urination. c. poor urine output. d. nausea.

B

A nurse is caring for a client who is about to begin taking epoetin. An increase in which of the following laboratory values should indicate to the nurse that the therapy is effective? A. WBC B. Hgb C.PT D.INR

B Epoetin, an erythropoietic growth factor, increases the production of RBCs for clients who have anemia due to chronic renal failure for chemotherapy. Hgb and Hct Should increase with effective therapy.

A nurse is monitoring for post operative complications in a client who had a kidney biopsy. Which of the following complications causes the most immediate risk to the client? A. Infection B. Hemorrhage C. Hematuria D. Kidney failure

B (Hemorrhage is the most immediate client risk following a kidney biopsy if clotting does not occur at the puncture site.)

An assessment finding indicating to the nurse that a 70-kg patient in septic shock is progressing to MODS includes A. respiratory rate of 10 breaths/min. B. fixed urine specific gravity at 1.010. C. MAP of 55 mm Hg. D. 360-ml urine output in 8 hours.

B A fixed urine specific gravity points to an inability of the kidney to concentrate urine caused by acute tubular necrosis. With MODS, the patient's respiratory rate would initially increase. The MAP of 55 shows continued shock, but not necessarily progression to MODS. A 360-ml urine output over 8 hours indicates adequate renal perfusion.

The nurse understands that the client with pernicious anemia will have which distinguishing laboratory findings? A. elevated Schilling's test B. absent intrinsic factor C. Sedimentation rate of 16 mm/hour D. RBCs 5.0 million

B An elevated excretion of the injected radioactive vitamin B12, which is protocol for the first and second stage of the Schilling test, indicates that the client has the intrinsic factor and can absorb vitamin B12 into the intestinal tract. A sedimentation rate of 16 mm/hour is normal for both men and women and is a nonspecific test to detect the presence of inflammation

An ileal conduit is created for a client after a radical cystectomy. Which of the following would the nurse expect to include in the client's plan of care? A. Exercises to promote sphincter control B. Application of an ostomy pouch C. Irrigating the urinary diversion D. Intermittent catheterizations

B An ileal conduit involves care of a urinary stoma, much like that of a fecal stoma, including the application of an ostomy pouch, skin protection, and stoma care. Intermittent catheterizations and irrigations are appropriate for a continent urinary diverse such as a Kock or Indiana pouch. Exercises to promote sphincter control are appropriate for an ureterosigmoidoscopy.)

A nurse is administering epoetin intravenously to a client who has renal failure. Which of the following actions should the nurse take? A. Save the next vial for the next dose B. Administer via IV bolus over 1 to 3 min C. Shake the vial before using D. dilute the drug first with D5w

B Answer: Administer via IV bolus over 1 to 3 min.: Instructions for administering the drug include administering it VIA IV bolus over 1 to 3 minutes.

A nurse is preparing to administer a prescribed dose of desmopressin to a client who has hemophilia A. The client's laboratory results indicate that the client has a sodium level of 130 mEq/L. Which of the following actions should the nurse take? A. Administer the drug with an analgesic B. Clarify the prescription with the provider C. Administer the required orally D. assess factor IX levels

B Answer: Clarify the prescription with the provider: Hyponatremia and fluid retention can occur with the administration of desmopressin, an antidiuretic hormone used in the treatment of hemophilia A. The client's sodium level is below the expected range of 136-145 mEq/L. The nurse should notify the provider of the client's current sodium level and clarify the prescription prior to administration.

A nurse is caring for a client who has hemophilia A and is about to begin taking desmopressin to prevent bleeding. The nurse should monitor the client for which of the following adverse reactions? A. Weight loss B. Edema C. polyuria D. Bradycardia

B Answer: an antidiuretic hormone, can cause fluid retention and edema. The nurse should monitor fluid intake and output for clients receiving this drug.

Which information about a patient who is receiving vasopressin (Pitressin) to treat septic shock is most important for the nurse to communicate to the healthcare provider? A. The patient's heart rate is 108 beats/min. B. The patient is complaining of chest pain. C. The patient's peripheral pulses are weak. D. The patient's urine output is 15 mL/hr.

B Because vasopressin is a potent vasoconstrictor, it may decrease coronary artery perfusion. The other information is consistent with the patient's diagnosis and should be reported to the health care provider but does not indicate a need for a change in therapy.

The nurse devises a teaching plan for the patient with aplastic anemia. Which of the following is the most important concept to teach for health maintenance? A. Eat animal protein and dark leafy vegetables each day B. Avoid exposure to others with acute infection C. Practice yoga and meditation to decrease stress and anxiety D. Get 8 hours of sleep at night and take naps during the day

B Clients with aplastic anemia are severely immunocompromised and at risk for infection and possible death related to bone marrow suppression and pancytopenia. Strict aseptic technique and reverse isolation are important measures to prevent infection

A client with pernicious anemia asks why she must take vitamin B12 injections for the rest of her life. What is the nurse's best response? A."The reason for your vitamin deficiency is an inability to absorb the vitamin because the stomach is not producing sufficient acid." B."The reason for your vitamin deficiency is an inability to absorb the vitamin because the stomach is not producing sufficient intrinsic factor." C."The reason for your vitamin deficiency is an excessive excretion of the vitamin because of kidney dysfunction." D."The reason for your vitamin deficiency is an increased requirement for the vitamin because of rapid red blood cell production."

B Most clients with pernicious anemia have deficient production of intrinsic factor in the stomach. Intrinsic factor attaches to the vitamin in the stomach and forms a complex that allows the vitamin to be absorbed in the small intestine. The stomach is producing enough acid, there is not an excessive excretion of the vitamin, and there is not a rapid production of RBCs in this condition.

A 78-year-old man has confusion and temperature of 104° F (40° C). He is a diabetic with purulent drainage from his right heel. After an infusion of 3 L of normal saline solution, his assessment findings are BP 84/40 mm Hg; heart rate 110; respiratory rate 42 and shallow; CO 8 L/minute; and PAWP 4 mm Hg. This patient's symptoms are most likely indicative of: A. sepsis. B. septic shock. C. multiple organ dysfunction syndrome. D. systemic inflammatory response syndrome.

B Septic shock is the presence of sepsis with hypotension despite fluid resuscitation along with the presence of inadequate tissue perfusion. To meet the diagnostic criteria for sepsis, the patient's temperature must be higher than 100.9° F (38.3° C), or the core temperature must be lower than 97.0° F (36° C). Hemodynamic parameters for septic shock include elevated heart rate; decreased pulse pressure, blood pressure, systemic vascular resistance, central venous pressure, and pulmonary artery wedge pressure; normal or elevated pulmonary vascular resistance; and decreased, normal, or increased pulmonary artery pressure, cardiac output, and mixed venous oxygen saturation.

When caring for a patient who has septic shock, which assessment finding is most important for the nurse to report to the health care provider? a. BP 92/56 mm Hg b. Skin cool and clammy c. Apical pulse 118 beats/min d. Arterial oxygen saturation 91%

B Since patients in the early stage of septic shock have warm and dry skin, the patient's cool and clammy skin indicates that shock is progressing. The other information also will be reported, but does not indicate deterioration of the patient's status.

Your patient's blood pressure is 72/56, heart rate 126, and respiration 24. The patient has a fungal infection in the lungs. The patient also has a fever, warm/flushed skin, and is restless. You notify the physician who suspects septic shock. You anticipate that the physician will order what treatment FIRST? A. Low-dose corticosteroids B. Crystalloids IV fluid bolus C. Norepinephrine D. 2 units of Packed Red Blood Cells

B The first treatment in regards to helping maintain tissue perfusion is fluid replacement with either crystalloid or colloid solutions. THEN vasopressors like Norepinephrine are ordered if the fluids don't help.

Your patient, who is post-op from a kidney transplant, has developed septic shock. Which statement below best reflects the interventions you will perform for this patient? A. Administer Norepinephrine before attempting a fluid resuscitation. B. Collect cultures and then administer IV antibiotics. C. Check blood glucose levels before starting any other treatments. D. Administer Drotrecogin Alpha within 48-72 hours.

B This is the only correct option. Option A is wrong because fluids are administered first, and if they don't work vasopressors (Norepinephrine) is administered. Option C is wrong because although blood glucose levels should be measured, it does not take precedence over other treatments. Option D is wrong because Drotrecogin alpha should be given within 24-48 hours of septic shock to be the most effective.

A patient with cardiogenic shock has a blood pressure of 70/38. In addition, the patient is experiencing dyspnea with a respiratory rate of 32 breaths per minute and has an oxygen saturation of 82% on room air. On auscultation, you note crackles throughout the lung fields. You notify the physician. What order below would you ask for an order clarification? A. Dopamine IV stat B. Normal saline IV bolus stat C. Furosemide IV stat D. Place patient on CPAP

B This patient with cardiogenic shock is experiencing a decrease in cardiac output (hence the blood pressure), so an order for Dopamine can help provide a positive inotropic effect (increase the contractility of the heart which will increase stroke volume and cardiac output). The patient is also experiencing pulmonary congestion due to the cardiogenic shock. The heart is failing to pump blood forward, so it is backing up in the lungs. This is leading to an increased respiratory rate, dyspnea, and low oxygen saturation. The order for Furosemide (which is a diuretic) will help remove the extra fluid volume from the lungs and the CPAP (continuous positive airway pressure) will help with oxygenation. The nurse would question the order for a normal saline IV bolus. This bolus would add more fluid to the lungs and further congest the fluids.

What is the BEST position for a patient in anaphylactic shock? A. Lateral recumbent B. Supine with legs elevated C. High Fowler's D. Semi-Fowler's

B This position will increase venous return to the heart, which will help increase cardiac output and blood pressure.

A patient is on IV Norepinephrine for treatment of septic shock. Which statement is FALSE about this medication? A. "The nurse should titrate this medication to maintain a MAP of 65 mmHg or greater." B. "This medication causes vasodilation and decreases systemic vascular resistance." C. "It is used when fluid replacement is not unsuccessful." D. "It is considered a vasopressor."

B This statement is FALSE because this medication causes vasoconstriction (not vasodilation) which INCREASES systemic vascular resistance.

Which diagnostic test can detect destruction of circulating platelets as the cause of thrombocytopenia? A. Hemoglobin B. Bone marrow analysis C. Prothrombin time (PT) D. Peripheral blood smear

B When destruction of circulating platelets is the cause, bone marrow analysis shows megakaryocytes (precursors of platelets) to be normal or increased, even though circulating platelets are reduced. Prothrombin time (PT) is used to assess secondary hemostasis. Peripheral blood smear is used to distinguish acquired disorders from congenital disorders. Hemoglobin measures the amount of protein in the red blood cells that carries oxygen to the body's organs and tissues and transports carbon dioxide from the organs and tissues back to the lungs.

A patient is diagnosed with septic shock. As the nurse you know this is a __________ form of shock. In addition, you're aware that __________ and _________ are also this form of shock. A. obstructive; hypovolemic and anaphylactic B. distributive; anaphylactic and neurogenic C. obstructive; cardiogenic and neurogenic D. distributive; anaphylactic and cardiogenic

B eptic shock is a form of distributive shock. This means there is an issue with the distribution of blood flow in the small blood vessels of the body. This results in a diminished supply of blood to the body's tissues and organs. Anaphylactic and neurogenic shock are also a type of distributive form of shock. Septic shock isn't occurring due to an issue with cardiac output, which occurs in hypovolemic and cardiogenic shock.

Which of the following is not a function of hypertonic solutions? A. maintains hydration B. promote waste elimination in kidneys C. treat hypovolemia D. expand volume

B this is a hypotonic function

A client has been prescribed nitrofurantoin (Macrodantin) for treatment of a lower urinary tract infection. Which of the following instructions should the nurse include when teaching the client how to take this medication? Select all that apply. A. "Take the medication on an empty stomach." B. "Your urine may become brown in color." C. "Increase your fluid intake." D. "Take the medication until your symptoms subside." E. "Take the medication with an antacid to decrease gastrointestinal distress."

B, C

A patient is being treated for cardiogenic shock. Which statement below best describes this condition? Select all that apply: A. "The patient will experience an increase in cardiac output due to an increase in preload and afterload." B. "A patient with this condition will experience decreased cardiac output and decreased tissue perfusion." C. "This condition occurs because the heart has an inadequate blood volume to pump." D. "Cardiogenic shock leads to pulmonary edema."

B, C

Select all the patients who are at MOST risk for iron-deficiency anemia: A 55 year old male who reports taking Ferrous Sulfate regularly. B. A 25 year old female who was recently diagnosed with Celiac Disease. C. A 35 year old female who is 36 weeks pregnant that reports craving ice. D. A 67 year old female with a Hemoglobin level of 14.

B, C

Which of the following benzos would be appropriate to give to an older adult client with a seizure disorder? Select all that apply A. diazepam B. alprazolam C. lorazepam D. clonazepam E. chlordiazepoxide

B, C

You're working on a neuro unit. Which of your patients below are at risk for developing neurogenic shock? Select all that apply: A. A 36-year-old with a spinal cord injury at L4. B. A 42-year-old who has spinal anesthesia. C. A 25-year-old with a spinal cord injury above T6. D. A 55-year-old patient who is reporting seeing green halos while taking Digoxin.

B, C Any patient who has had a cervical or upper thoracic (above T6) spinal cord injury, receiving spinal anesthesia, or taking drugs that affect the autonomic or sympathetic nervous system is at risk for developing neurogenic shock.

A patient with a fever is lethargic and has a blood pressure of 89/56. The patient's white blood cell count is elevated. The physician suspects the patient is developing septic shock. What other findings indicate this patient is in the "early" or "compensated" stage of septic shock? Select all that apply: A. Urinary output of 60 mL over 4 hours B. Warm and flushed skin C. Tachycardia D. Bradypnea

B, C In the early or compensated stage of septic shock, the patient is in a hyperdynamic state. This is different from the other types of shock like hypovolemic or cardiogenic (vasoconstriction is occurring in these types of shock). In septic shock, vasodilation is occurring and this leads to WARM and FLUSHED skin in the early stage. However, in the late stage the skin will be cool and clammy. Tachycardia and TACHYpnea (not bradypnea) occurs in the early stage too as a compensatory mechanism. Oliguria (option A) is in the late stage or uncompensated when the kidneys are starting to fail.

A client is brought to the emergency department for increasing weakness and muscle twitching. The laboratory results include a potassium level of 7.0 mEq/L (7.0 mmol/L). Which assessments does the nurse make? SELECT ALL THAT APPLY. A. History of liver disease B. Use of salt substitute C. Use of an ACE inhibitor D. Potassium-sparing diuretics E. Prescription for insulin

B, C, D, RATIONALE: When caring for an ED client with an elevated potassium level, the nurse needs to assess the client for any use of salt substitutes, any use of ACE inhibitors or potassium-sparing diuretics, as well as kidney disease.History of liver disease does not increase the client's potassium level. Insulin, which moves potassium into the cell, can be used as a treatment for hyperkalemia, in addition to diabetes. Taking insulin would lower the potassium level.

The nurse is caring for a client receiving lactated Ringer's solution IV for rehydration. Which assessments will the nurse monitor during intravenous therapy? SELECT ALL THAT APPLY A. Blood serum glucose B. Blood pressure C. Pulse rate and quality D. Urinary output E. Urine specific gravity

B, C, D, E RATIONALE: The two most important areas to monitor during rehydration are pulse rate and quality and urine output. In addition, decreasing specific gravity of urine is also an indication of rehydration. Blood pressure is another important vital sign to monitor during rehydration.Blood glucose changes do not have a direct relation to a client's hydration status; lactated ringers are free from glucose.

A patient with a severe infection has developed septic shock. The patient's blood pressure is 72/44, heart rate 130, respiration 22, oxygen saturation 96% on high-flow oxygen, and temperature 103.6 'F. The patient's mean arterial pressure (MAP) is 53 mmHg. Based on these findings, you know this patient is experiencing diminished tissue perfusion and needs treatment to improve tissue perfusion to prevent organ dysfunction. In regards to the pathophysiology of septic shock, what is occurring in the body that is leading to this decrease in tissue perfusion? Select all that apply: A. Absolute hypovolemia B. Vasodilation C. Increased capillary permeability D. Increased systemic vascular resistance E. Clot formation in microcirculation F. A significantly decreased cardiac output

B, C, E Septic shock occurs due to sepsis. Sepsis is the body's reaction to an infection and will lead to septic shock if this reaction is not treated. This reaction is the activation of the body's inflammatory system, but it's MAJORLY amplified and system wide. Cardiac output is not the problem in septic shock as with other types of shocks like hypovolemic or cardiogenic. CO is actually high or normal during the early stages of septic shock. It only decreases to the end of septic shock when heart function fails. The issue is with what is going on beyond the heart in the vessels. Substances are released by the microorganism that has invaded the body. This causes the immune system to release substances that will cause system wide vasodilation of the vessels (this will cause a DECREASE in systemic vascular resistance, blood to pool, and this decreases blood flow to the organs/tissues) along with an increase in capillary permeability (this causes fluid to leave the intravascular system and depletes the circulatory system of fluid and further decreases blood flow to the organs/fluids...this is RELATIVE (not absolute) hypovolemia). Furthermore, clots will form in the microcirculation due to plasma activating factor being released. This will cause platelets to aggregate and block blood flow even more to the organs/tissues. All of this will lead to decreased tissue perfusion and deprive cells of oxygen.

A​ high-acuity client with acute kidney injury is at risk for developing hypernatremia. Which collaborative nursing action addresses this​ risk? (Select all that​ apply.) A. Administering bicarbonate infusion B. Administering maintenance​ 0.45% saline IV solution C. Assessing extremities for edema D. Maintaining intake and output balance E. Monitoring ECG for arrhythmias

B, D

You're assessing your patient with cardiogenic shock, what signs and symptoms do you expect to find in this condition? Select all that apply: A. Warm, flushed skin B. Prolonged capillary refill C. Urinary output >30 mL/hr D. Systolic blood pressure <90 mmHg E. Crackles in lung fields F. Dyspnea D. Decreased BUN and creatinine G. Strong peripheral pulses H. Chest pain

B, D, E, F, and H. Signs and symptoms of cardiogenic shock will be related to LOW cardiac output and decreased perfusion to organs/tissues. Capillary refill will be prolonged >2 seconds, urinary output will be <30 mL/hr, systolic blood pressure will be <90 mmHg, pulmonary edema will present with fluid in the lungs (hence crackles in the lungs), dyspnea, and chest pain (due to decreased blood flow to the heart muscle).

A nurse is providing discharge teaching for a client who has chronic kidney disease (CKD). Which of the following statements by the client indicates an understanding of the teaching? A. "I will consume foods high in protein." B. "I will decrease my intake of foods high in phosphorus." C. "I will limit my intake of foods high in calcium." D. "I will add salt to the foods I consume."

B. "I will decrease my intake of foods high in phosphorus." Clients who have CKD should limit the intake of foods high in phosphorus due to the decrease in the kidneys' ability to excrete it.

A nurse is providing education regarding cyclosporine for a client who had a kidney transplant 2 days ago. Which of the following statements by the nurse is appropriate? A. "You may experience hair loss due to the medication therapy you'll be taking." B. "You will need to continue taking this medication to protect your new kidneys." C. "Use an over-the-counter anti-inflammatory medication for aches and pains." D. "You will be at an increased risk for infection if you stop taking this medication."

B. "You will need to continue taking this medication to protect your new kidneys." Clients must take cyclosporine daily for the life of the transplanted organ.

A client is diagnosed with septic shock. Which collaborative therapy should the nurse consider to be the priority​? A. Administering vasoactive drugs B. Administering intravenous​ (IV) fluids C. Administering an inotropic drug D. Administering IV antibiotics

B. Administering intravenous​ (IV) fluids Fluid replacement is the priority intervention to help improve tissue perfusion. The client will also be immediately placed on oxygen to treat hypoxia. These interventions will be followed by administration of a​ broad-spectrum antibiotic. When fluid replacement alone is not sufficient to reverse​ shock, vasoactive drugs​ (drugs causing vasoconstriction or​ vasodilation) and inotropic drugs​ (drugs improving cardiac​ contractility) may be administered.

A nurse is teaching a client who has a new diagnosis of acute pyelonephritis. Which of the following instructions should the nurse include in the teaching? A. Drink up to 1,500 mL of fluid per day. B. Avoid the use of NSAIDs for pain. C. Monitor peripheral blood glucose level twice per day. D. Increase dietary protein intake.

B. Avoid the use of NSAIDs for pain. The nurse should instruct the client to avoid the use of NSAIDs for pain, which can further damage the kidney.

A client develops fluid overload while in the intensive care unit. Which nursing intervention does the nurse perform first? A. Draws blood for laboratory tests B. Elevates the head of the bed C. Places the extremities in a dependent position D. Puts the client in a side-lying position

B. Elevates the head of the bed The nurse first needs to elevate the client's head of bed when caring for a client with fluid overload. Remember to follow the ABC's and perform interventions that promote lung expansion and oxygenation to relieve symptoms of fluid overload.Drawing blood for laboratory tests may be indicated, but would not be performed first. Placing the extremities in a dependent position increases peripheral edema, and positioning the client in a side-

A nurse is reviewing the laboratory report of a client who has fluid volume excess. Which of the following laboratory values should the nurse expect? a. Hemoglobin 20 g/dL b. Hematocrit 34% c. BUN 25 mg/dL d. Urine specific gravity 1.050

B. Hematocrit 34% This hematocrit level is below the expected reference range. A 2+ pitting edema indicates fluid overload, which can cause hemodilution and a decreased hematocrit.

Which clinical manifestation would lead the nurse to suspect that a client is experiencing hypermagnesemia? A. Muscle pain and acute rhabdomyolysis B. Hot, flushed skin and diaphoresis C. Soft-tissue calcification and hyperreflexia D. Increased respiratory rate and depth

B. Hot, flushed skin and diaphoresis Hypermagnesemia is manifested by hot, flushed skin and diaphoresis. The client also may exhibit hypotension, lethargy, drowsiness, and absent deep tendon reflexes

A nurse is obtaining a voided urine culture and sensitivity for a client who has manifestations of a urinary tract infection. Which of the following actions should the nurse take? A. Collect the client's urine in a clean specimen container. B. Instruct the client to initiate the flow of urine before collecting the specimen. C. Obtain the client's first morning voiding on the following day. D. Place the client's urine specimen in a container with a preservative.

B. Instruct the client to initiate the flow of urine before collecting the specimen The nurse should instruct the client to pass a sterile container into the urine stream after initiating the flow of urine.

A nurse is planning care for a client who has a serum potassium level of 3.0 mEq/L. The nurse should plan to monitor the client for which of the following findings? a. Hyperactive deep-tendon reflexes b. Orthostatic hypotension c. Rapid, deep respirations d. Strong, bounding pulse

B. Orthostatic hypotension Hypokalemia can lead to hypotension. The nurse should monitor the client for orthostatic hypotension.

A nurse working in a women's health clinic is caring for a client who reports urinary urgency and dysuria. Which of the following additional findings should the nurse identify as an indication of a urinary tract infection (UTI)? A. Vaginal discharge B. Pyuria C. Glucosuria D. Elevated creatine kinase-MB

B. Pyuria The nurse should identify pyuria, which is white blood cells in the urine, as a common manifestation of a UTI.

A nurse is providing teaching for a client who has urge urinary incontinence. The nurse should include which of the following instructions? A. Sit on the toilet with water running every 4 hr. B. Set an interval for toileting based on previous voiding pattern. C. Respond immediately to the urge to void. D. Self-catheterize daily following a regular voiding.

B. Set an interval for toileting based on previous voiding pattern. When the client can maintain continence, the length of time between voids is gradually increased. The nurse should establish a toileting schedule to coincide with the client's voiding pattern. The nurse can gradually increase the length of time between voids as the client maintains continence.

A positive Janus-Kinase 2 gene is indicative of what disorder? A. immune thrombocytopenia B. polycythemia vera C. aplastic anemia D. pernicious anemia

B. polycythemia vera

A 36 year old female, who is 29 weeks pregnant, reports she is experiencing burning when voiding. The physician orders a urinalysis. Which statement by the patient demonstrates she understands how to collect the specimen? A. "I'll hold the cup firmly against the urethra while collecting the sample." B. "I will cleanse back to front with the antiseptic wipe before peeing in the cup." C. "First, I will pee a small amount of urine in the toilet and then collect the rest in the cup." D. "I will be sure to drink a lot of fluids to keep the urine diluted before peeing into the cup."

C

A 74-year-old patient is extremely confused and does not respond to commands or stimulation. The patient respiratory rate is 28 and labored, oxygen saturation 86%, heart rate 120, blood pressure 70/40, mean arterial pressure is 50 mmHg, and temperature is 97 'F. The patient's heart rhythm is atrial fibrillation. The patient's urinary output is 5 mL/hr. The patient's labs: blood pH 7.15, serum lactate 15 mmol/L, BUN 55 mg/dL, Creatinine 6 mg/dL. In addition, the patient is now starting to have slight oozing of blood around puncture sites. Based on this scenario, what stage of shock is this patient most likely experiencing? A. Initial B. Proliferative C. Progressive D. Compensatory

C

A nurse is assessing laboratory values for a client who may have acute glomerulonephritis. Which of the following findings should the nurse report to the provider? A. Urine specific gravity of 1.022 B. BUN of 16 mg/dL C. Creatinine clearance of 48 mL/min/m2 D. Potassium level of 4.2 mEq/L

C

A nurse is providing teaching to a client who has a prescription for trimethoprim/sulfamethoxazole. Which of the following instructions should the nurse include in the teaching? A. Take the medication without food B. Expect a fine, red rash as a transient effect C. Drink 8 to 10 glasses of water daily D. Store the medication in the refrigerator

C

A patient comes into the emergency room for dehydration and severe nausea. What delegation made by the graduate nurse requires the charge nurse to intervene? A. "It could help to try and eat slowly." B. "Eating plain, bland toast may help with digestion." C. "Heating foods up or drinking while hot is known to be beneficial for rehydrating patients with nausea." D. "Avoid drinking while eating at the same time, as it can make you feel more bloated or full."

C

The client was dx with iron-deficiency anemia is prescribed ferrous gluconate orally. Which should the nurse teach the client? A. Take Imodium, and anti diarrheal, OTC for diarrhea B. Limit exercise for several weeks until a tolerance is achieved C. The stools may be very dark, and this can mask blood D. Eat only red meats and organ meats for protein

C

The nurse identifies which medication as posing a significant risk of causing confusion, somnolence, psychosis, and visual disturbances in elderly patients? A. Metronidazole [Flagyl] B. Rifampin [Rifadin] C. Ciprofloxacin [Cipro] D. Daptomycin [Cubicin

C

The nurse is instructing the client with chronic renal failure to maintain adequate nutritional intake. Which diet would be most appropriate? a. high-carbohydrate, high-protein b. high-calcium, high-potassium, high-protein c. low-protein, low-sodium, low-potassium d. low-protein, high-potassium

C

A​ 63-year-old man is admitted with postrenal acute kidney injury​ (AKI) because of a kidney stone. Vascular volume and renal perfusion have been restored and he is on fluid restriction. During the past 24​ hours, he has voided 250 mL of urine. He has not had any other type of output. How much fluid should the client receive over the next 24​ hours? A. 2750 mL B. 1250 mL C. 850 mL D. 3000 mL

C Fluid intake is calculated for these clients by adding the amount of output for the previous 24 hours to 600 mL to allow for insensible losses. The​ client's output for the past 24 hours was 250​ mL; added to 600​ mL, the fluid volume calculation equals 850 mL

The nurse provides education to a patient with thrombocytopenia about precautions to be taken at home. The nurse identifies that further teaching is needed when the patient performs which action? A. Drinks 14 glasses of water daily B. Shaves once a day using an electric razor C. Uses an alcohol based mouthwash twice a day D. Brushes teeth with a soft-bristle toothbrush twice a day

C A patient with thrombocytopenia should not use alcohol-based mouthwashes, because alcohol-based mouthwashes will dry the mouth and increase bleeding. Patients with thrombocytopenia should drink plenty of fluids to prevent constipation. Patients with thrombocytopenia should shave using an electric razor, not blades, to prevent the risk for injury. Patients with thrombocytopenia should use a soft-bristle toothbrush to prevent gum injury.

A nurse is monitoring a client following ferrous sulfate administration. The nurse should monitor the client for which of the following side effects? A. Phlebitis B. Dark, orange colored stools C. Constipation D. Injection site pain

C Answer: C Oral Iron supplementation is associated with constipation. The nurse should encourage the client to consume adequate amounts of fiber and fluids in their diet to minimize this effect.

A patient with neurogenic shock is experiencing a heart rate of 30 bpm. What medication does the nurse anticipate will be ordered by the physician STAT? A. Adenosine B. Warfarin C. Atropine D. Norepinephrine

C Atropine will quickly increase the heart rate and block the effects of the parasympathetic system on the body.

They physician orders a Dobutamine IV drip on a patient in cardiogenic shock. After starting the IV drip, the nurse would make it priority to monitor for? A. Rebound hypertension B. Ringing in the ears C. Worsening hypotension D. severe headache

C Dobutamine increases contractility and cardiac output, BUT causes vasodilation due to the way it acts on receptors and this may make hypotension WORSE. The patient may be started on norepinephrine or dopamine if worsening of hypotension occurs.

A patient is in anaphylactic shock. The patient has a severe allergy to peanuts and mistakenly consumed an eggroll containing peanut ingredients during his lunch break. The patient is given Epinephrine intramuscularly. As the nurse, you know this medication will have what effect on the body? A. It will prevent a recurrent attack. B. It will cause vasoconstriction and decrease the blood pressure. C. It will help dilate the airways. D. It will help block the effects of histamine in the body.

C Epinephrine acts as a vasopressor and will actually dilate the airway. Epinephrine performs vasoconstriction which will INCREASE the blood pressure. It does not prevent a recurrent attack (corticosteroids may help with this), and it does not block the effects of histamine (antihistamine helps with this).

Your patient is having a sudden and severe anaphylactic reaction to a medication. You immediately stop the medication and call a rapid response. The patient's blood pressure is 80/52, heart rate 120, and oxygen saturation 87%. Audible wheezing is noted along with facial redness and swelling. As the nurse you know that the first initial treatment for this patient's condition is? A. IV Diphenhydramine B. IV Normal Saline Bolus C. IM Epinephrine D. Nebulized Albuterol

C IM or subQ Epinephrine is the first-line treatment for anaphylaxis. Epinephrine will cause vasoconstriction (this will increase the blood pressure and decrease swelling) and bronchodilation (this will dilate the airways). This patient's cardiovascular and respiratory system is compromised. Therefore, epinephrine will provide fast relief with anaphylaxis.

The nurse reviews a patient's medical record and suspects heparin-induced thrombocytopenia (HIT). Which finding supports the nurse's conclusion? A. Patient has a viral infection. B. Patient has hemolytic anemia. C. Patient has a platelet count of 100,000/µL. D. Patient has systemic lupus erythematosus.

C Long-term heparin therapy can causes heparin-induced thrombocytopenia (HIT) that results in decrease in the platelet count. A platelet count of 100,000/µL indicates HIT. Immune thrombocytopenic purpura (ITP) will be suspected if the patient has a viral infection. Thrombotic thrombocytopenic purpura (TTP) will be suspected if the patient has hemolytic anemia and an autoimmune disorder like systemic lupus erythematosus.

What is the treatment for a septic patient with elevations of serum lactate? A. packed RBC's B. norepinephrine C. isotonic fluid administration D. blood cultures

C PRBC's won't help a septic patient norepinephrine would be given if fluids didn't work you'd want to draw blood cultures but it is not the priority

The nurse corrects the nursing student when caring for a client with neutropenia secondary to chemotherapy in which circumstance? A. The student scrubs the hub of IV tubing before administering an antibiotic. B. The nurse overhears the student explaining to the client the importance of handwashing. C. The student teaches the client that symptoms of neutropenia include fatigue and weakness. D. The nurse observes the student providing oral hygiene and perineal care.

C Symptoms of neutropenia include low neutrophil count, fever, and signs and symptoms of infection; the student should be corrected. Asepsis with IV lines is an appropriate action. Handwashing is an essential component of client care, especially when the client is at risk for neutropenia. Hygiene and perineal care help prevent infection and sepsis.

A nurse is assessing a client for Chvostek's sign. Which of the following techniques should the nurse use to perform this test? A. Apply a blood pressure cuff to the client's arm. B. Place the stethoscope bell over the client's carotid artery. C. Tap lightly on the client's cheek. D. Ask the client to lower her chin to her chest.

C The nurse taps the client's cheek over the facial nerve just below and anterior to the ear to elicit Chvostek's sign. A positive response is indicated when the client exhibits facial twitching on this side of her face.

The nurse is treating a client with a serum potassium level of​ 6.7mEq/L who is already on restricted potassium intake. Which medication may be ordered to reduce the neuromuscular effects of this increased serum​ level? A. Antibiotic B. H2-receptor antagonist C. Calcium chloride D. Lactated Ringer

C With significant​ hyperkalemia, calcium​ chloride, bicarbonate, and insulin and glucose may be given intravenously to reduce serum potassium levels by moving potassium into the cells

A nurse is caring for a client who has a prescription for chlorothiazide to treat hypertension. The nurse should plan to monitor the client for which of the following adverse effects? A. Thrombophlebitis B. Hyperactive reflexes C. Muscle weakness D. Hypoglycemia

C chlorothiazide promotes the excretion of potassium, which can cause muscle weakness

A nurse is preparing to administer amphotericin B IV to a client who has a systemic fungal infection. Which of the following drugs should the nurse prepare to administer prior to the infusion to prevent or minimize adverse reactions? (Select all that apply). A. Aspirin B. Hydrocortisone C. Acetaminophen D. Diphenhydramine E. Ibuprofen

C, D

The nurse caring for a patient after urinary diversion surgery monitors the patient closely for peritonitis by assessing for which of the following? Select all that apply. A. Hyperactive bowel sounds B. Muscle flaccidity C. Leukocytosis D. Abdominal distention

C, D

Which statements are INCORRECT about the compensatory stage of shock. Select all that apply: A. This stage is reversible. B. During this stage blood is shunted away from the kidneys, lungs, skin, and gastrointestinal system to the brain and heart. C. During this stage blood flow to the kidneys is reduced, which causes the kidneys to activate the renin-angiotensin system, and this will lead to major vasodilation to the arterial and venous system. D. One hallmark sign of this stage is that there is an increase in capillary permeability. E. A patient is at risk for a paralytic ileus during this stage.

C, D These options reflect INCORRECT statements about the compensatory stages and all the other stages are TRUE about this stage. Option C is wrong because although the kidneys activate the RAS, this does NOT lead to vasodilation but VASOCONSTRICTION to the arterial and venous system. Option D is wrong because this is a hallmark sign in the PROGRESSIVE stage not compensatory.

Which diagnostic test result would the nurse expect to find for a client with septic​ shock? (Select all that​ apply.) A. Normal WBC count B. Decreased blood urea nitrogen​ (BUN) and creatinine levels C. Decreased serum pH level D. Increased neutrophil count E. Increased partial pressure of​ oxygen, arterial ​(PaO2​)

C,D C. Decreased serum pH level D. Increased neutrophil count Septic shock causes a decrease in pH​ (indicating acidosis), a decrease in PaO2 and total oxygen​ saturation, and an increase in PaCO2. WBC count decreases as cells are​ destroyed, and increased neutrophils and monocytes indicate acute bacterial infection. Renal function declines as reduced perfusion and microclotting damage the small renal arterioles. As perfusion of the kidneys is decreased and renal function is​ reduced, the BUN and creatinine levels​ increase, as do urine specific gravity and osmolality.

The nurse is evaluating the discharge teaching for a client who has an ileal conduit. Which of the following statements indicates that the client has correctly understood the teaching? Select all that apply. A:"If I limit my fluid intake I will not have to empty my ostomy pouch as often." B:"I can place an aspirin tablet in my pouch to decrease odor." C:"I can usually keep my ostomy pouch on for 3 to 7 days before changing it." D:"I must use a skin barrier to protect my skin from urine." E:"I should empty my ostomy pouch of urine when it is full."

C,D: "I can usually keep my ostomy pouch on for 3 to 7 days before changing it.", "I must use a skin barrier to protect my skin from urine." The client with an ileal conduit must learn self-care activities related to care of the stoma and ostomy appliances. The client should be taught to increase fluid intake to about 3,000 ml per day and should not limit intake. Adequate fluid intake helps to flush mucus from the ileal conduit. The ostomy appliance should be changed approximately every 3 to 7 days and whenever a leak develops. A skin barrier is essential to protecting the skin from the irritation of the urine. An aspirin should not be used as a method of odor control because it can be an irritant to the stoma and lead to ulceration. The ostomy pouch should be emptied when it is one-third to one-half full to prevent the weight from pulling the appliance away from the skin.

The patient in the oliguric phase of AKI excreted 300 mL of urine in addition to 100 mL of other losses during the past 24 hours. With appropriate calculations, you determine that for the next 24 hours the patient's fluid allocation is A. 600 mL B. 800 mL C. 1000 mL D. 1200 mL

C. 1000 mL Fluid intake must be closely monitored during the oliguric phase. The rule for calculating the fluid restriction is to add all losses for the previous 24 hours to 600 mL for insensible losses.

A nurse is caring for a client who has received hemodialysis. The nurse should identify that which of the following findings places the client at risk for seizures? A. Hypokalemia B. A rapid increase of catecholamines C. A rapid decrease in fluid D. Hypercalcemia

C. A rapid decrease in fluid A rapid decrease in fluid can result in cerebral edema and increased intracranial pressure, placing the client at risk for seizures.

The nurse caring for a client diagnosed with urosepsis finds spider angiomas of the extremities and cool fingertips. The healthcare provider suspects disseminated intravascular coagulation​ (DIC). Which collaborative intervention should the nurse​ implement? A. Give total parenteral nutrition. B. Decrease intravenous​ (IV) fluids. C. Administer​ fresh-frozen plasma. D. Administer insulin.

C. Administer​ fresh-frozen plasma. A client with DIC is bleeding and clotting at the same time.​ Therefore, to control the​ bleeding, the healthcare provider would prescribe fresh frozen​ plasma, which contains clotting factors. Insulin is used to manage blood glucose levels in clients with diabetes. The client would receive IV fluids at a higher rate to compensate for fluid shifts. Total parenteral nutrition is used to treat​ malnutrition, not DIC

A nurse in a provider's office receives a call from a client who is taking amoxicillin to treat a respiratory infection and reports a rash and wheezing. Which of the following instructions should the nurse give the client? A. Wait 1 hr and contact the provider if there is no improvement. B. Skip today's dose of amoxicillin and resume taking the drug tomorrow. C. Call emergency services immediately. D. Take an NSAID to reduce skin and airway inflammation.

C. Call emergency services immediately.

A nurse is assessing a client who has chronic kidney disease and has completed her third peritoneal dialysis (PD) treatment. Which of the following findings should the nurse report to the provider? A. Greater outflow of dialysate than inflow B. Weight loss C. Cloudy dialysate effluent D. Report of pain during inflow

C. Cloudy dialysate effluent Cloudy or opaque drainage is an early manifestation of peritonitis. The nurse should notify the provider immediately because infection can be a life-threatening complication.

A client who has been diagnosed with bladder cancer is scheduled for an ileal conduit. Preoperatively, the nurse reinforces the client's understanding of the surgical procedure by explaining that an ileal conduit A. Is a temporary procedure that can be reversed later. B. Diverts urine into the sigmoid colon, where it is expelled through the rectum. C. Conveys urine from the ureters to a stoma opening in the abdomen. D. Creates an opening in the bladder that allows urine to drain into an external pouch

C. Conveys urine from the ureters to a stoma opening in the abdomen. An ileal conduit is a permanent urinary diversion in which a portion of the ileum is surgically resected and one end of the segment is closed. The ureters are surgically attached to this segment of the ileum, and the open end of the ileum is brought to the skin surface on the abdomen to form the stoma. The client must wear a pouch to collect the urine that continually flows through the conduit. The bladder is removed during the surgical procedure and the ileal conduit is not reversible.

A nurse is performing an admission assessment of a client who has acute glomerulonephritis. The nurse should expect which of the following findings? A. Low blood pressure B. Polyuria C. Dark-colored urine D. Weight loss

C. Dark-colored urine The client who has acute glomerulonephritis usually has urine that is a dark, reddish-brown color.

A nurse is caring for a client who has chronic kidney failure and the following laboratory results: BUN 196 mg/dL, sodium 152 mEq/L, and potassium 7.3 mEq/L. Which of the following interventions should the nurse implement? A. Initiate an IV infusion of 0.9% sodium chloride. B. Give oral spironolactone. C. Infuse regular insulin in dextrose 10% in water. D. Administer furosemide.

C. Infuse regular insulin in dextrose 10% in water. The client who has an elevated potassium level should receive regular insulin with dextrose 10% in water by continuous IV infusion to facilitate moving potassium out of the extracellular fluid into intracellular fluid.

The nurse is assessing a client with a sodium level of 118 mEq/L (118 mmol/L). Which activity takes priority? A. Monitoring urine output B. Encouraging sodium rich fluids and foods throughout the day C. Instructing the client not to ambulate without assistance D. Assessing deep tendon reflexes

C. Instructing the client not to ambulate without assistance RATIONALE: Safety is the priority in this instance. Instructing the client not to ambulate without assistance is the priority for a client with a sodium level of 118 mEq/L (118 mmol/L). This sodium level denotes severe hyponatremia which makes depolarization slower and cell membranes less excitable. This is manifested as general muscle weakness which is worse in the legs and arms. Additionally, this client may have developed confusion from cerebral edema.Monitoring urine output needs to be done but is not the priority action in this situation. Generally, fluid is restricted, rather than sodium rich foods offered, to minimize the hyponatremia. While the nurse may assess muscle strength and deep tendon reflex responses, safety is the priority.

Furosemide (Lasix) has been ordered for a client with heart failure, shortness of breath, and 3+ pitting edema of the lower extremities. Which assessment finding indicates to the nurse that the medication has been effective? A. The client's potassium level is 5.1 mEq/L (5.1 mmol/L). B. The client's heart rate is 101 beats per minute. C. The client is free from adventitious breath sounds. D. The client has experienced a weight gain of 1 pound (0.5 kg).

C. The client is free from adventitious breath sounds. RATIONALE: The nurse recognizes that Furosemide is effective when the client is free from adventitious breath sounds such as crackles. Other positive outcomes to the diuretic include normal heart rate, weight loss with resolution of edema, and increased urine output.A potassium value of 5.1 mEq/L or (5.1 mmol/L) is normal. Changes in potassium levels such as hypokalemia are side effects of furosemide, not therapeutic effects. Although a fall in the client's BP may occur with the decrease in body fluid, this is not the priority. Tachycardia may occur during episodes of fluid volume excess or deficit and does not directly indicate the medication has been effective. Weight loss, rather than weight gain, is often the effect of Furosemide, caused by the diuresis.

A nurse is caring for a client who is about to begin gentamicin therapy to treat an infection. The nurse should monitor the client for an alteration in which of the following? A. Bowel function B. Peripheral pulses C. Urine output D. Level of consciousness

C. Urine output Gentamicin, an aminoglycoside, can cause nephrotoxicity. The nurse should monitor the client's BUN and creatine levels and for an increased output of diluted urine. It is also essential to monitor serum gentamicin levels and maintain a therapeutic range.

Nursing interventions for a patient with hyponatremia include: A. administering hypotonic IV fluids B. encouraging water intake C. restricting fluid intake D. restricting sodium intake

C. restricting fluid intake

A nurse is assessing a client who has a serum calcium level of 8.1 mg/dL. Which of the following findings is the priority for the nurse to assess? a. Deep-tendon reflexes b. Cardiac rhythm c. Peripheral sensation d. Bowel sounds

Cardiac rhythm When using the airway, breathing, circulation approach to client care, the nurse should determine that assessing the cardiac rhythm is the priority. Calcium levels below the expected reference range can cause ECG changes, bradycardia, or tachycardia.

A nurse is assessing a client who has respiratory acidosis. Which of the following findings should the nurse except? a. Confusion b. Peripheral edema c. Facial flushing d. Hyperreflexia

Confusion A client who has respiratory acidosis will experience confusion from a lack of cerebral perfusion. If acidosis is not reversed, the client's level of consciousness will decrease and coma may occur. Facial flushing and warmth are manifestations of metabolic acidosis. Pale, cyanotic, dry skin is a manifestation of respiratory acidosis as ineffective breathing causes a lack of perfusion to the tissues. Hyporeflexia, not hyperreflexia, is a manifestation of respiratory acidosis. As acidosis increases, hyperkalemia can occur, causing muscle weakness, flaccid paralysis, and hyporeflexia.

A client with iron deficiency anemia is scheduled for discharge. Which instruction about prescribed ferrous gluconate therapy should the nurse include in the teaching plan? A. "Take the medication with an antacid." B. "Take the medication with a glass of milk." C. "Take the medication with cereal." D. "Take the medication on an empty stomach."

D

A nurse is administering Ciprofloxacin and Phenazopyridine to a client who has a severe urinary tract infection. The client asks why both medications are needed. Which of the following responses should the nurse make? A. Phenazopyridine decreases the adverse effects of ciprofloxacin hydrochloride. B. Combining Phenazopyridine with ciprofloxacin hydrochloride shortens the course of therapy. C. The use of Phenazopyridine allows the doctor to prescribe a lower dosage of ciprofloxacin hydrochloride. D. Ciprofloxacin hydrochloride treats the infection, and Phenazopyridine treats pain.

D

A nurse is providing teaching to a client who is to have an x-ray of the kidneys, ureters, and bladders (KUB). Which of the following statements should the nurse include in the teaching? A. "Contrast dye is given during the procedure." B. "An enema is necessary before the procedure." C. "You will need to lie in a prone position during the procedure." D. "The procedure determines whether a kidney stone is present."

D

A patient is being admitted with dehydration due to nausea and vomiting. Which fluid would you expect the patient to be started on? A. 5% Dextrose in 0.9% Saline B. 0.33% saline C. 0.225% saline D. 0.9% Normal Saline

D

Hypertonic dehydration can be described as: A. The equal loss of water and electrolytes that results in loss of circulating volume B. Fluid moving into interstitial space causing volume deficit and cells swelling C. Shift of fluid from vascular space to a part of the body that is not easily exchanged with the rest of the extracellular fluid D. Fluid moving out of interstitial space causing cellular dehydration

D

Patient's blood pressure is: 220/118 What is the patient's MAP? A. 99 mmHg B. 36 mmHg C. 89 mmHg D. 152 mmHg

D

Patient's blood pressure is: 80/56 What is the patient's MAP and how do you interpret this finding? A. 78 mmHg, normal B. 96 mmHg, low C. 86 mmHg, normal D. 64 mmHg, low

D

To prepare for a patient's transfusion of packed red blood cells, the nurse should select which intravenous solution to use for the procedure? A. 3% normal saline B. Lactated Ringer's C. 5% dextrose in water D. 0.9% normal saline

D

What indicates to the nurse that a patient with AKI is in the recovery phase? a. a return to normal weight b. a urine output of 3,700 mL/day c. decreasing sodium and potassium levels d. decreasing BUN and creatinine levels

D

When caring for a patient during the oliguric phase of AKI, which nursing intervention is appropriate? A. weight the pt 3x weekly B. increase dietary K and Na C. provide low protein, high carb diet D. restrict fluids according to previous daily loss

D

Which of the following is a drug with modest antihypertensive effects that is often reserved for use in patients that develop hypokalemia in response to taking another more potent diuretic? A. furosemide B. hydrochlorothiazide C. indapamide D. triamterene

D

A nurse is changing the stoma appliance on a patient's ileal conduit. Which characteristic of the stoma would alert the nurse that the patient is experiencing ischemia? a. The stoma is hard and dry. b. The stoma is a pale pink color. c. The stoma is swollen. d. The stoma is a purple-blue color.

D A purple-blue stoma may reflect compromised circulation or ischemia. A pale stoma may indicate anemia. The stoma may be swollen at first, but that condition should subside with time. A normal stoma should be moist and dark pink to red in color.

Which of the following is the best definition for anasarca? A. flushed facial congestion seen in the second stage of septic shock B. episodic burning of the hands and feet C. inflamed lips seen in the second stage of shock D. profound edema that is a manifestation in the third progressive stage of septic shock

D A. NO - plethora is flushed facial congestion seen in polycythemia B. NO - erythromelalgia seen in polycythemia C. NO - cheilitis seen in iron deficiency anemia

A nurse is caring for a client who is taking ferrous sulfate to treat iron deficiency anemia and develops toxicity. Which of the following drugs should the nurse expect to use to treat this complication? A. Flumazenil B.Acetylcysteine C.Naloxone D.Deferoxamine

D Answer: Deferoxamine: Indications of iron toxicity include nausea, vomiting and diarrhea. Iron toxicity can lead to acidosis and shock. A chelating agent, such as deferoxamine, binds to the iron to reduce toxicity.

A nurse is caring for a client who is about to begin factor VIII therapy to treat hemophilia A. When administering factor VIII, which of the following actions should the nurse take? A. Administer the powdered form orally B. Premedication with aspirin C. Administer it via rapid IV bolus D. Have emergency equipment ready

D Answer: Have emergency equipment ready: Factor VIII can cause a hypersensitive reaction and anaphylaxis. The nurse should monitor the client for hives, fever, wheezing, and difficulty breathing and have emergency equipment and drugs readily available.

Which client situation requires the nurse to discuss the importance of avoiding foods high in potassium? A. 14-year-old Elena who is taking diuretics B. 16-year-old John Joseph with ileostomy C. 16-year-old Gabriel with metabolic alkalosis D. 18-year-old Albert who has renal disease

D Clients with renal disease are predisposed to hyperkalemia and should avoid foods high in potassium. Clients receiving diuretics, with ileostomies, or with metabolic acidosis may be hypokalemic and should be encouraged to eat foods high in potassium.

A patient in septic shock receives large amounts of IV fluids. However, this was unsuccessful in maintaining tissue perfusion. As the nurse, you would anticipate the physician to order what NEXT? A. IV corticosteroids B. Vasopressin C. Dobutamine D. Norepinephrine

D Fluids are ordered FIRST in septic shock. If this is unsuccessful, then vasopressors are ordered NEXT. Norepinephrine is used as a first-line agent. Dobutamine may sometimes be used but for its inotropic effects on the heart.

The nurse is determining a​ client's fluid balance. Which method should the nurse use to identify this​ client's fluid volume excess or​ deficit? A. Blood pressure B. Intake and output C. Skin turgor D. Daily weight

D Rationale: Daily weight is the best indicator of fluid volume excess or deficit

What positively charged cation is found abundantly outside of the body cells? A. Potassium (K) B. Phosphorous (P) C. Magnesium (M) D. Sodium (Na)

D Sodium is found abundantly in the extracellular fluid (outside of the cell) and potassium is found abundantly in the intracellular fluid (inside the cell).

The nurse is preparing to give a dose of trimethoprim-sulfamethoxazole (TMP-SMX) and learns that the patient takes warfarin (Coumadin). The nurse will request an order for a. a decreased dose of TMP-SMX. b. a different antibiotic. c. an increased dose of warfarin. d. coagulation studies.

D Sulfonamides can increase the anticoagulant effects of warfarin. The nurse should request INR levels.

Khaleesi is admitted in the hospital due to having lower than normal potassium level in her bloodstream. Her medical history reveals vomiting and diarrhea prior to hospitalization. Which foods should the nurse instruct the client to increase? A. Whole grains and nuts B. Milk products and green, leafy vegetables C. Pork products and canned vegetables D. Orange juice and bananas

D The client with hypokalemia needs to increase the intake of foods high in potassium. Orange juice and bananas are high in potassium, along with raisins, apricots, avocados, beans, and potatoes. Whole grains and nuts would be encouraged for the client with hypomagnesemia; milk products and green, leafy vegetables are good sources of calcium for the client with hypocalcemia. Pork products and canned vegetables are high in sodium and are encouraged for the client with hyponatremia.

A client with anemia asks, "Why am I feeling tired all the time?" How does the nurse respond? A. "How many hours are you sleeping at night?" B. "You are not getting enough iron." C. "You need to rest more when you are sick." D. "Your cells are delivering less oxygen than you need."

D The single most common symptom of anemia is fatigue. This problem occurs because oxygen delivery to cells is less than is required to meet normal oxygen needs.

A client comes into the health clinic 3 years after undergoing a resection of the terminal ileum complaining of weakness, shortness of breath, and a sore tongue. Which client statement indicates a need for intervention and client teaching? A. "I have been drinking plenty of fluids." B. "I have been gargling with warm salt water for my sore tongue." C. "I have 3 to 4 loose stools per day." D. "I take a vitamin B12 tablet every day."

D Vitamin B12 combines with intrinsic factor in the stomach and is then carried to the ileum, where it is absorbed in the bloodstream. In this situation, vitamin B12 cannot be absorbed regardless of the amount of oral intake of sources of vitamin B12 such as animal protein or vitamin B12 tablets. Vitamin B12 needs to be injected every month, because the ileum has been surgically removed.

During shock, when a patient experiences a drop in cardiac output, the body tries to compensate by stimulating the sympathetic nervous system, which causes the release of _________ and ________. This will lead to? A. acetylcholine and dopamine, vasodilation B. epinephrine and norepinephrine, vasodilation C. dopamine and epinephrine, vasoconstriction D. norepinephrine and epinephrine, vasoconstriction

D When a patient in shock experiences a drop in blood pressure (due to a drop in cardiac output), the baroreceptors (in the carotid and aortic arch) will sense this and stimulate the sympathetic nervous system. The SNS will release the neurotransmitters NOREPINEPHRINE and EPINEPRHINE and this will lead to VASOCONSTRICTION. By causing constriction of the vessels, it will temporarily increase cardiac output by increase the blood pressure and heart rate, which will in turn increase perfusion to cells.

A patient arrived to the clinic complaining of generalized pruritus and pain in the extremities. He has a history of diabetes, HTN, dyslipidemia, COPD, and A-fib. His blood pressure is 155/90. His face is a ruddy color. On palpation of his abdomen, his spleen is palpable. His blood values indicate a Hgb of 18.5 and Hct of 60%. The nurse suspects which diagnosis? A. Polycythemia Vera B. Multiple Myeloma C. Thrombocytopenia D. Secondary Polycythemia

D based on a history of COPD, the nurse suspects that the polycythemia is based on the secondary cause of chronic reduced oxygen intake. Other possible causes of secondary polycythemia would include smoking history, cyanotic heart disease, or even living in a high altitude. There are also several other causes. If a cause is noted, polycythemia Vera (primary polycythemia) is ruled out

A client with iron deficiency anemia is scheduled for discharge. Which instruction about prescribed ferrous gluconate therapy should the nurse include in the teaching plan? A."Take the medication with an antacid." B."Take the medication with a glass of milk." C."Take the medication with cereal." D."Take the medication on an empty stomach."

D however can be taken with Vitamin c - can stain teeth - turns stools brown

A nurse is preparing a teaching plan for a male client who has a continent internal ileal reservoir following surgery to treat bladder cancer. Which of the following statements should the nurse include in the teaching plan? A. "This should not affect your ability to have sexual intercourse." B. "You should empty your new bladder when it feels full." C. "You will need to avoid foods that produce intestinal gas." D. "You must insert a catheter through your stoma to drain the urine."

D. "You must insert a catheter through your stoma to drain the urine." The client must use intermittent catheterization to drain urine from the continent internal ileal reservoir

A nurse is planning care for a client who is postoperative following a nephrectomy. Which of the following assessments is the priority for the nurse to evaluate? A. Bowel sounds B. WBC count C. Pain level D. Blood pressure

D. Blood Pressure The greatest risk to the client is acute adrenal insufficiency. The adrenal gland can be removed or damaged during nephrectomy. The nurse should evaluate the client for hypotension, decreased urine output, and decreased level of consciousness.

The nurse is assessing a client for septic shock. Which assessment finding supports this​ diagnosis? A. Pulse, 60​ beats/min B. Blood​ pressure, 110/72 mmHg C. Oxygen​ saturation, 92% D. Central venous pressure​ (CVP), 1 mmHg

D. Central venous pressure​ (CVP), 1 mmHg When assessing a client with​ sepsis, the nurse monitors the​ client's hemodynamic status with a CVP intravenous line or pulmonary artery catheter. Normal CVP is 2-8 mmHg and is decreased with septic shock. The other vital signs are within normal limits.

A nurse is caring for a client the night before a scheduled intravenous urography. Which of the following is the nurse's priority intervention? A. Inform the client about dietary limitations. B. Place the informed consent document in the client's record. C. Administer a bowel preparation to the client. D. Determine if the client has an allergy to iodine or shellfish.

D. Determine if the client has an allergy to iodine or shellfish. The greatest risk to the client is injury or death from an allergic reaction to radiopaque contrast media. The nurse should determine if the client has an allergy to iodine or shellfish, which indicates the client is at high risk of having an allergic reaction to the contrast media

A client with hypermagnesemia is seen in the emergency department (ED). Which of these interventions is most appropriate? A. Monitor for hyperactive reflexes B. prepare for endotracheal intubation C. Institute teaching on avoiding magnesium rich foods D. Place the client on a cardiac monitor

D. Place the client on a cardiac monitor RATIONALE: Hypermagnesemia causes changes in cardiac rhythm and may result in cardiac arrest, therefore instituting cardiac monitoring is most appropriate.Reflexes are typically reduced in the presence of hypermagnesemia. There is no indication that the client has signs and symptoms of respiratory distress at this time, however the nurse would monitor the client for respiratory weakness and respiratory failure. The nurse will institute teaching after the emergency passes and the cause of the magnesium excess is determined.

A nurse is caring for a client who is taking ciprofloxacin to treat a urinary tract infection. The client also takes prednisolone to treat rheumatoid arthritis. Recognizing the adverse effects of ciprofloxacin, the nurse should instruct the client to report which of the following adverse effects? A. Tachycardia B. Hair loss C. Insomnia D. Tendon pain

D. Tendon pain

A nurse is assessing a client who has hyperkalemia. Which of the following findings should the nurse expect? a. Decreased muscle strength b. Decreased gastric motility c. Increased heart rate d. Increased blood pressure

Decreased muscle strength Hyperkalemia can cause muscle weakness. The nurse should monitor the client's muscle strength.

The health care provider prescribes these actions for a patient who has possible septic shock with a BP of 70/42 mm Hg and oxygen saturation of 90%. In which order will the nurse implement the actions? A. Obtain blood and urine cultures. B. Give vancomycin (Vancocin) 1 g IV. C. Infuse vasopressin (Pitressin) 0.01 units/min. D. Administer normal saline 1000 mL over 30 minutes. E. Titrate oxygen administration to keep O2 saturation >95%.

E, D, C, A, B The initial action for this hypotensive and hypoxemic patient should be to improve the oxygen saturation, followed by infusion of IV fluids and vasopressors to improve perfusion. Cultures should be obtained before administration of antibiotics.

While reviewing a client's laboratory results, a nurse notes a serum calcium level of 0.8 mg/dL. Which of the following actions should the nurse take? a. Implement seizure precautions. b. Administer phosphate. c. Initiate diuretic therapy. d. Prepare the client for hemodialysis.

Implement seizure precautions The client is at risk for seizures due to low excitation threshold as a result of the client's decreased calcium level. The nurse should initiate seizure precautions to prevent injury.

Match the solution with each category. A. Normal saline (0.9% NS) B. Half normal saline (0.45% NS) C. Lactated Ringers D. D5W E. D10W F. 3% Sodium Chloride G. D5 NaCl • Isotonic solution • Hypotonic solution • Hypertonic solution

Isotonic solution • A, C, D Hypotonic solution • B, D Hypertonic solution • E, F, G

A nurse is assessing a client who has dehydration. Which of the following assessments is the priority? a. Skin turgor b. Urine output c. Weight d. Mental status

Mental status The greatest risk to this client is injury from declining mental status or a fall from worsened dehydration. Therefore, assessing the client's mental status is the priority.

A nurse is caring for a client who reports difficulty breathing and tingling in both hands. His respiratory rate is 36/min and he appears very restless. Which of the following values should the nurse anticipate to be outside the expected reference range if the client is experiencing respiratory alkalosis? a. PaO2 b. PaCO2 c. Sodium d. Bicarbonate

PaCo2 With respiratory alkalosis, the PaCO2 level is decreased.

A nurse is caring for a client who requires continuous cardiac monitoring. The nurse identifies a prolonged PR interval and a widened QRS complex. Which of the following laboratory values supports this finding? a. Sodium 152 mEq/L b. Chloride 102 mEq/L c. Magnesium 1.8 mEq/L d. Potassium 6.1 mEq/L

Potassium 6.1 mEq/L Hyperkalemia can cause a prolonged PR interval; a wide QRS complex; flat or absent P waves; and tall, peaked T waves.

A nurse is assessing a client who is using PCA following a thoracotomy. The client is short of breath, appears restless, and has a respiratory rate of 28/min. The client's ABG results are pH 7.52, PoO2 89 mm hg, and HCO3- 24 mEq/L. Which of the following actions should the nurse take? a. Instruct the client to cough forcefully. b. Assist the client with ambulation. c. Provide calming interventions. d. Discontinue the PCA.

Provide calming interventions The client's respiratory rate is above the expected range. Calming the client should decrease the respiratory rate, which will cause the client's carbon dioxide levels to increase. This will help correct the pH imbalance.

A nurse is admitting a client who has status asthmaticus. The client's ABG results are pH 7.32, PaO2 74 mmhg, PaC02 56 mm hg, and HCO3- 26 mEq/L. The nurse should interpret these laboratory values as which of the following imbalances. a. Respiratory acidosis b. Respiratory alkalosis c. Metabolic acidosis d. Metabolic alkalosis

Respiratory acidosis Status asthmaticus causes inadequate gas exchange, resulting in a low pH and PaO2, an elevated PaCO2, and an HCO3- within the expected reference range. These laboratory values indicate respiratory acidosis.

A nurse is reviewing the medical record of a client who had diabetes mellitus and is receiving regular insulin by continuous IV infusion to treat diabetic ketoacidosis. Which of the following findings should the nurse report to the provider? a. Urine output of 30 mL/hr b. Blood glucose of 180 mg/dL c. Serum potassium 3.0 mEq/L d. BUN 18 mg/dL

Serum potassium 3.0 mEq/L This serum potassium level is outside the expected reference range. The nurse should report this finding to the provider.

A nurse is evaluating a client who is receiving IV fluids to treat isotonic dehydration. Which of the following laboratory findings indicates that the fluid therapy has been effective? a. BUN 26 mg/dL b. Serum sodium 142 mEq/L c. Hct 56% d. Urine specific gravity 1.035

Serum sodium 142 mEq/L Isotonic dehydration includes loss of water and electrolytes due to a decrease in oral intake of water and salt. A serum sodium level of 142 mEq/L is within the expected reference range and indicates that the fluid therapy has been effective.BUN is elevated. HCT is elevated and USG is elevated.

A nurse is assessing a client who has a phosphorus level of 2.4 mg/dL. Which of the following findings should the nurse expect? a. Hepatic failure b. Abdominal pain c. Slow peripheral pulses d. Increase in cardiac output

Slow peripheral pulses Hypophosphatemia causes slow peripheral pulses that are difficult to detect and can eventually result in cardiac muscle damage.

A nurse is assessing a client who is receiving hydrochlorothiazide and notes that the client is confused and lethargic. Which of the following laboratory values should the nurse report to the provider? a. Sodium 128 mEq/L b. Potassium 4.8 mEq/L c. Calcium 9.1 mg/dL d. Magnesium 2.0 mEq/L

Sodium 128 mEq/L This level is below the expected reference range and is the likely cause of the client's altered mental status. The nurse should report this finding to the provider and monitor the client for weakened respiratory effort.

A nurse is preparing to administer oral potassium for a client who has a potassium level of 5.5 mEq/L. Which of the following actions should the nurse take first? a. Administer a hypertonic solution. b. Repeat the potassium level. c. Withhold the medication. d. Monitor for paresthesia.

Withhold the medication The greatest risk to this client is injury from hyperkalemia. Therefore, the priority action is to withhold the oral potassium and notify the provider.

A nurse is caring for a client who is experiencing respiratory distress as a result of pulmonary edema. Which of the following actions should the nurse take first? a. Assist with intubation. b. Initiate high-flow oxygen therapy. c. Administer a rapid-acting diuretic. d. Provide cardiac monitoring.

b. Initiate high-flow oxygen therapy The priority action the nurse should take when using the airway, breathing, circulation approach to client care is to administer high-flow oxygen therapy by face mask at 5 to 6 L/min to keep the client's oxygen saturation above 90%.

A nurse is caring for a client who has a urinary tract infection (UTI). Which of the following is the priority intervention by the nurse? a. offer a warm sitz bath b. recommend drinking cranberry juice c. encourage increased fluids d. administer antibiotics

d. administer antibiotics D: The greatest risk to the client is injury to the renal system and sepsis from the UTI. The priority intervention is to administer antibiotics

More Sepsis Questions

https://quizlet.com/427975892/sepsis-practice-questions-flash-cards/

A nurse is reviewing the ABG results for four clients. Which of the following findings should the nurse identify as metabolic acidosis? a. pH 7.51, PaO2 94 mm Hg, PaCO2 38 mm Hg, HCO3- 29 mEq/L b. pH 7.48, PaO2 89 mm Hg, PaCO2 30 mm Hg, HCO3- 24 mEq/L c. pH 7.36, PaO2 77 mm Hg, PaCO2 52 mm Hg, HCO3- 26 mEq/L d. pH 7.26, PaO2 84 mm Hg, PaCO2 38 mm Hg, HCO3- 20 mEq/L

pH 7.26, Pa02 84mm hg, PaC02 38 mmhg, HCO3- 20 mEq/L When pH and HCO3- are both above or below the expected reference range, a metabolic imbalance is present. A pH of 7.26 indicates acidosis and a HCO3- of 20 mEq/L indicates the acidosis is due to a metabolic cause. Therefore, the nurse should identify these findings as metabolic acidosis.


Kaugnay na mga set ng pag-aaral

Graphing Polynomial Functions Instruction

View Set

PHI3626 Obojobo - Feminism and Feminist Ethics

View Set

PEDs Chapt 17 Nursing Care of the Child with a Disorder of the Eyes or Ears

View Set

6th Grade Bible - Unit 3 - John the Baptist, The Birth and Childhood of Jesus

View Set

Social studies open-ending answer key

View Set

Systems Phys Homework (for Final)

View Set

Salts: acidic, basic, or neutral

View Set